Did the new image of black hole confirm the general theory of relativity? Announcing the arrival of Valued Associate #679: Cesar Manara Planned maintenance scheduled April 17/18, 2019 at 00:00UTC (8:00pm US/Eastern)Why isn't the circumferential light around the M87 black hole's event horizon symmetric?How can Quasars emit anything if they're black holes?How can cosmic jets exist?How did the first image of a black hole test the general relativity?Does GR provide a maximum electric field limit?From where (in space-time) does Hawking radiation originate?Falling into a black holeIf nothing in the universe can travel faster than light, how come light can't escape a black hole?Does cosmic censorship rule out stable toroidal black holes? How?Connection between the Big Bang and Black HolesHow to obtain initial conditions to image Kerr black hole?Observer inside event horizon of an extremely large black holeBased on our current observations, what all prevents a formed black from experiencing a repulsive force to overcome its gravitational force…?How did the first image of a black hole test the general relativity?

Problem when applying foreach loop

Losing the Initialization Vector in Cipher Block Chaining

What do you call a plan that's an alternative plan in case your initial plan fails?

What was the last x86 CPU that did not have the x87 floating-point unit built in?

Did the new image of black hole confirm the general theory of relativity?

Is above average number of years spent on PhD considered a red flag in future academia or industry positions?

The following signatures were invalid: EXPKEYSIG 1397BC53640DB551

Statistical model of ligand substitution

How can I make names more distinctive without making them longer?

How to say that you spent the night with someone, you were only sleeping and nothing else?

How does modal jazz use chord progressions?

When is phishing education going too far?

What's the difference between (size_t)-1 and ~0?

Estimate capacitor parameters

What do you call the holes in a flute?

What to do with post with dry rot?

How can I protect witches in combat who wear limited clothing?

Why use gamma over alpha radiation?

What is the largest species of polychaete?

Can I throw a sword that doesn't have the Thrown property at someone?

Writing Thesis: Copying from published papers

Why is there no army of Iron-Mans in the MCU?

Unable to start mainnet node docker container

Is it possible to ask for a hotel room without minibar/extra services?



Did the new image of black hole confirm the general theory of relativity?



Announcing the arrival of Valued Associate #679: Cesar Manara
Planned maintenance scheduled April 17/18, 2019 at 00:00UTC (8:00pm US/Eastern)Why isn't the circumferential light around the M87 black hole's event horizon symmetric?How can Quasars emit anything if they're black holes?How can cosmic jets exist?How did the first image of a black hole test the general relativity?Does GR provide a maximum electric field limit?From where (in space-time) does Hawking radiation originate?Falling into a black holeIf nothing in the universe can travel faster than light, how come light can't escape a black hole?Does cosmic censorship rule out stable toroidal black holes? How?Connection between the Big Bang and Black HolesHow to obtain initial conditions to image Kerr black hole?Observer inside event horizon of an extremely large black holeBased on our current observations, what all prevents a formed black from experiencing a repulsive force to overcome its gravitational force…?How did the first image of a black hole test the general relativity?










25












$begingroup$


How can we do it just by looking at the image. But I heard in news saying "Einstein was right! black hole image confirms GTR. The image is so less detailed that I can't even make some pretty good points. Please correct me if I'm wrong on any aspect. Please provide a link if this question sounds duplicate..










share|cite|improve this question











$endgroup$







  • 5




    $begingroup$
    Read the science explained at the official website; it should answer your questions: eventhorizontelescope.org/science. News is not the best place to go to for science.
    $endgroup$
    – Avantgarde
    2 days ago






  • 8




    $begingroup$
    Re, "...just by looking at the image." You can't learn much just by looking at that image. But if you use the theory to predict what the picture should look like, and then you take the picture and it agrees with your prediction, then that ought to boost your confidence in the theory.
    $endgroup$
    – Solomon Slow
    2 days ago






  • 1




    $begingroup$
    Einstein was right about it being a beautiful theory only a few people understand. The problem with experimental data that are recorded from vast distances in space is that there will always be some noise in the data, and filtering out the noise is going to be a problem because then whatever type of filtering you might use will affect the actual incoming data. By affect, I mean that the filtering algorithms used, might have an unintended effect on it, making it align with GR..
    $endgroup$
    – Natural Number Guy
    2 days ago






  • 1




    $begingroup$
    @SolomonSlow Actually, given the scale of the image, you can by eye pretty much identify the two main features predicted by GR.
    $endgroup$
    – Rob Jeffries
    yesterday










  • $begingroup$
    I am glad that this question is not closed as "lack of research " or "unclear of what you are asking" or "too broad" or "duplicate ", or else we won't get the excellent answers
    $endgroup$
    – Graviton
    yesterday
















25












$begingroup$


How can we do it just by looking at the image. But I heard in news saying "Einstein was right! black hole image confirms GTR. The image is so less detailed that I can't even make some pretty good points. Please correct me if I'm wrong on any aspect. Please provide a link if this question sounds duplicate..










share|cite|improve this question











$endgroup$







  • 5




    $begingroup$
    Read the science explained at the official website; it should answer your questions: eventhorizontelescope.org/science. News is not the best place to go to for science.
    $endgroup$
    – Avantgarde
    2 days ago






  • 8




    $begingroup$
    Re, "...just by looking at the image." You can't learn much just by looking at that image. But if you use the theory to predict what the picture should look like, and then you take the picture and it agrees with your prediction, then that ought to boost your confidence in the theory.
    $endgroup$
    – Solomon Slow
    2 days ago






  • 1




    $begingroup$
    Einstein was right about it being a beautiful theory only a few people understand. The problem with experimental data that are recorded from vast distances in space is that there will always be some noise in the data, and filtering out the noise is going to be a problem because then whatever type of filtering you might use will affect the actual incoming data. By affect, I mean that the filtering algorithms used, might have an unintended effect on it, making it align with GR..
    $endgroup$
    – Natural Number Guy
    2 days ago






  • 1




    $begingroup$
    @SolomonSlow Actually, given the scale of the image, you can by eye pretty much identify the two main features predicted by GR.
    $endgroup$
    – Rob Jeffries
    yesterday










  • $begingroup$
    I am glad that this question is not closed as "lack of research " or "unclear of what you are asking" or "too broad" or "duplicate ", or else we won't get the excellent answers
    $endgroup$
    – Graviton
    yesterday














25












25








25


5



$begingroup$


How can we do it just by looking at the image. But I heard in news saying "Einstein was right! black hole image confirms GTR. The image is so less detailed that I can't even make some pretty good points. Please correct me if I'm wrong on any aspect. Please provide a link if this question sounds duplicate..










share|cite|improve this question











$endgroup$




How can we do it just by looking at the image. But I heard in news saying "Einstein was right! black hole image confirms GTR. The image is so less detailed that I can't even make some pretty good points. Please correct me if I'm wrong on any aspect. Please provide a link if this question sounds duplicate..







general-relativity black-holes astronomy






share|cite|improve this question















share|cite|improve this question













share|cite|improve this question




share|cite|improve this question








edited yesterday







Liquid

















asked 2 days ago









LiquidLiquid

12826




12826







  • 5




    $begingroup$
    Read the science explained at the official website; it should answer your questions: eventhorizontelescope.org/science. News is not the best place to go to for science.
    $endgroup$
    – Avantgarde
    2 days ago






  • 8




    $begingroup$
    Re, "...just by looking at the image." You can't learn much just by looking at that image. But if you use the theory to predict what the picture should look like, and then you take the picture and it agrees with your prediction, then that ought to boost your confidence in the theory.
    $endgroup$
    – Solomon Slow
    2 days ago






  • 1




    $begingroup$
    Einstein was right about it being a beautiful theory only a few people understand. The problem with experimental data that are recorded from vast distances in space is that there will always be some noise in the data, and filtering out the noise is going to be a problem because then whatever type of filtering you might use will affect the actual incoming data. By affect, I mean that the filtering algorithms used, might have an unintended effect on it, making it align with GR..
    $endgroup$
    – Natural Number Guy
    2 days ago






  • 1




    $begingroup$
    @SolomonSlow Actually, given the scale of the image, you can by eye pretty much identify the two main features predicted by GR.
    $endgroup$
    – Rob Jeffries
    yesterday










  • $begingroup$
    I am glad that this question is not closed as "lack of research " or "unclear of what you are asking" or "too broad" or "duplicate ", or else we won't get the excellent answers
    $endgroup$
    – Graviton
    yesterday













  • 5




    $begingroup$
    Read the science explained at the official website; it should answer your questions: eventhorizontelescope.org/science. News is not the best place to go to for science.
    $endgroup$
    – Avantgarde
    2 days ago






  • 8




    $begingroup$
    Re, "...just by looking at the image." You can't learn much just by looking at that image. But if you use the theory to predict what the picture should look like, and then you take the picture and it agrees with your prediction, then that ought to boost your confidence in the theory.
    $endgroup$
    – Solomon Slow
    2 days ago






  • 1




    $begingroup$
    Einstein was right about it being a beautiful theory only a few people understand. The problem with experimental data that are recorded from vast distances in space is that there will always be some noise in the data, and filtering out the noise is going to be a problem because then whatever type of filtering you might use will affect the actual incoming data. By affect, I mean that the filtering algorithms used, might have an unintended effect on it, making it align with GR..
    $endgroup$
    – Natural Number Guy
    2 days ago






  • 1




    $begingroup$
    @SolomonSlow Actually, given the scale of the image, you can by eye pretty much identify the two main features predicted by GR.
    $endgroup$
    – Rob Jeffries
    yesterday










  • $begingroup$
    I am glad that this question is not closed as "lack of research " or "unclear of what you are asking" or "too broad" or "duplicate ", or else we won't get the excellent answers
    $endgroup$
    – Graviton
    yesterday








5




5




$begingroup$
Read the science explained at the official website; it should answer your questions: eventhorizontelescope.org/science. News is not the best place to go to for science.
$endgroup$
– Avantgarde
2 days ago




$begingroup$
Read the science explained at the official website; it should answer your questions: eventhorizontelescope.org/science. News is not the best place to go to for science.
$endgroup$
– Avantgarde
2 days ago




8




8




$begingroup$
Re, "...just by looking at the image." You can't learn much just by looking at that image. But if you use the theory to predict what the picture should look like, and then you take the picture and it agrees with your prediction, then that ought to boost your confidence in the theory.
$endgroup$
– Solomon Slow
2 days ago




$begingroup$
Re, "...just by looking at the image." You can't learn much just by looking at that image. But if you use the theory to predict what the picture should look like, and then you take the picture and it agrees with your prediction, then that ought to boost your confidence in the theory.
$endgroup$
– Solomon Slow
2 days ago




1




1




$begingroup$
Einstein was right about it being a beautiful theory only a few people understand. The problem with experimental data that are recorded from vast distances in space is that there will always be some noise in the data, and filtering out the noise is going to be a problem because then whatever type of filtering you might use will affect the actual incoming data. By affect, I mean that the filtering algorithms used, might have an unintended effect on it, making it align with GR..
$endgroup$
– Natural Number Guy
2 days ago




$begingroup$
Einstein was right about it being a beautiful theory only a few people understand. The problem with experimental data that are recorded from vast distances in space is that there will always be some noise in the data, and filtering out the noise is going to be a problem because then whatever type of filtering you might use will affect the actual incoming data. By affect, I mean that the filtering algorithms used, might have an unintended effect on it, making it align with GR..
$endgroup$
– Natural Number Guy
2 days ago




1




1




$begingroup$
@SolomonSlow Actually, given the scale of the image, you can by eye pretty much identify the two main features predicted by GR.
$endgroup$
– Rob Jeffries
yesterday




$begingroup$
@SolomonSlow Actually, given the scale of the image, you can by eye pretty much identify the two main features predicted by GR.
$endgroup$
– Rob Jeffries
yesterday












$begingroup$
I am glad that this question is not closed as "lack of research " or "unclear of what you are asking" or "too broad" or "duplicate ", or else we won't get the excellent answers
$endgroup$
– Graviton
yesterday





$begingroup$
I am glad that this question is not closed as "lack of research " or "unclear of what you are asking" or "too broad" or "duplicate ", or else we won't get the excellent answers
$endgroup$
– Graviton
yesterday











5 Answers
5






active

oldest

votes


















49












$begingroup$

I think it's fair to say that the EHT image definitely is consistent with GR, and so GR continues to agree with experimental data so far. The leading paper in the 10th April 2019 issue of Astrophysical Journal letters says (first sentence of the 'Discussion' section):




A number of elements reinforce the robustness of our image and the conclusion that it is consistent with the shadow of a black hole as predicted by GR.




I'm unhappy about the notion that this 'confirms' GR: it would be more correct to say that GR has not been shown to be wrong by this observation: nothing can definitively confirm a theory, which can only be shown to agree with experimental data so far.



This depends of course on the definition of 'confirm': above I am taking it to mean 'shown to be correct' which I think is the everyday usage and the one implied in your question, and it's that meaning I object to. In particular it is clearly not the case that this shows 'Einstein was right': it shows that GR agrees with experiment (extremely well!) so far, and this and LIGO both show (or are showing) that GR agrees with experiment in regions where the gravitational field is strong.



(Note that, when used informally by scientists, 'confirm' very often means exactly 'shown to agree with experiment so far' and in that sense GR has been confirmed (again) by this observation. I'm assuming that this is not the meaning you meant however.)




At least one other answer to this question is excellent and very much worth reading in addition to this.






share|cite|improve this answer











$endgroup$








  • 10




    $begingroup$
    It "confirms" it in the same way that the observation of gravitational lensing during the 1919 solar eclipse confirmed it.
    $endgroup$
    – Barmar
    2 days ago






  • 11




    $begingroup$
    @Barmar which is to say, it didn't disprove it. That's the best a theory can hope. "Confirm" is such an imprecise word that makes non-scientists get completely the wrong idea about the role of evidence in science.
    $endgroup$
    – Roman Starkov
    2 days ago






  • 1




    $begingroup$
    Yet it was good enough to make Einstein a household name. I think what makes some things more confirming is that the prediction they agree with is something quite unexpected according to previous theories. I'm not sure if the same thing could be said for the EHT pictures, because of all the evidence we've already found in the past century.
    $endgroup$
    – Barmar
    2 days ago






  • 2




    $begingroup$
    " it would be more correct to say that GR has not been shown to be wrong by this observation: nothing can definitively confirm a theory," Sorry, but you've got the terminology wrong. A theory is tested by comparing its predictions against experimental data. If the data does not agree with the theory, the theory is disproved (at least, provisionally). If the data does agree, the theory is confirmed (but not proven). You are confusing proof with confirmation.
    $endgroup$
    – WhatRoughBeast
    yesterday






  • 1




    $begingroup$
    @WhatRoughBeast: Isn't theory/hypothesis confusing the general public enough already? Is it really, absolutely necessary to invent another bit of jargon on top of that?
    $endgroup$
    – Kevin
    yesterday



















28












$begingroup$

Answer: The images from the Event Horizon Telescope (EHT) are consistent with what general relativity predicts. So if that's what the OP means by "confirm general relativity", then the answer is yes.



To appreciate the significance of the EHT images, we need to remember how science works. Theories are inspired by observations, but theories are not deduced from observations; we certainly cannot deduce general relativity from a single image. It's the other way around: theories make predictions. If the predictions consistently match what we observe, then we say that the theory works. Of course, contriving a theory that makes one prediction that matches one observation is trivial. Finding a theory that makes many predictions that match many observations is more challenging. General relativity is such a theory, and its agreement with this new observation — the images from the Event Horizon Telescope — is a nice addition to the large portfolio of general relativity's confirmed predictions.



Even though the black hole is enormous, it is so far away that the diameter of the imaged ring spans less than $20$ billionths of a degree ($<20times 10^-9$ degree) in the sky, so pristine resolution cannot be expected; the fact that they were able to resolve it at all is remarkable. Still, the image shows some general features that are consistent with what is expected from the light-bending effects associated with a rapidly spinning black hole in general relativity — not just any rapidly spinning black hole, but one whose size, mass, spin, and orientation are all consistent with other observations associated with that same black hole in the core of the galaxy M87.



A few of these observations are reviewed below, followed by comments on how the EHT images compare to predictions from general relativity.




Other observations: The jet



One of the most prominent associated observations is the jet emanating from the galaxy's core, shown here in images from the Hubble Space Telescope [$2$]:



enter image description here



To give a feeling for the scale of this picture, this is what hubblesite.org [$3$] says about the image:




At a distance of 50 million light-years, M87 is too distant for Hubble to discern individual stars. The dozens of star-like points swarming about M87 are, instead, themselves clusters of hundreds of thousands of stars each.




Here's another view of the jet, with scale-bars:



enter image description here



This image (from figure 2 in [$4$]) was made in 1999 using VLBI observations at a wavelength of 7 millimeters. The white dot marked $6r_S$ represents a circle with a diameter of $6$ times the alleged Schwarzschild radius. The scale bar marked "$1$ kpc" represents one kiloparsec, which is roughly 3000 light-years.



According to general relativity, a rapidly spinning black hole with an accretion disk can generate intense magnetic fields (but see [$5$]) that funnel material from the accreting plasma into a jet emanating along the black hole's axis of rotation. The fact that the observed jet is so straight over a distance of thousands of light-years implies that it must be produced by an engine that maintains a very consistent orientation for a time span of at least thousands of years, as a supermassive black hole is expected to do.




Other observations: The accretion disk



According to [$6$]:




HST [Hubble Space Telescope] imaged a disk of ionized gas, with a radius of $sim$ 50 pc [50 parsecs, roughly 150 light-years] centered on the galactic core... The high resolution of HST allowed the spectrum [which is sensitive to the Doppler effect] of this ionized gas to be measured as a function of position across the gas disk, thereby allowing the kinematics of the disk to be determined... It was found that the velocity profile of the central 20 pc of the gas disk possessed a Keplerian profile (i.e., $v propto r^-1/2$) as expected if the gas was orbiting in the gravitational potential of a point-like mass... The only known and long-lived object to possess such a large mass in a small region of space, and be as under-luminous as observed, is a SMBH [Super-Massive Black Hole].




In other words, these observations showed evidence for gas disk with the velocity profile that would be expected if it were orbiting a supermassive black hole. Note that the measured gas velocities on opposite sides of the central body differ from each other by roughly 1000 kilometers per second.




Other observations: The absense of strong surface emission



According to a report [$7$] published in 2015:




Observations at millimeter wavelengths with the Event Horizon Telescope have localized the emission from the base of this jet [shown above] to angular scales comparable to the putative black hole horizon. The jet might be powered directly by an accretion disk or by electromagnetic extraction of the rotational energy of the black hole. However, even the latter mechanism requires a confining thick accretion disk to maintain the required magnetic flux near the black hole. Therefore, regardless of the jet mechanism, the observed jet power in M87 implies a certain minimum mass accretion rate. If the central compact object in M87 were not a black hole but had a surface, this accretion would result in considerable thermal near-infrared and optical emission from the surface. Current flux limits on the nucleus of M87 strongly constrain any such surface emission. This rules out the presence of a surface and thereby provides indirect evidence for an event horizon.




Regarding why the event horizon of a black hole is expected to be so dim even though the intense fields generate a powerful jet, see these Physics SE posts:



  • How can Quasars emit anything if they're black holes?


  • How can cosmic jets exist?



Comparing the EHT images to predictions



Page 5 in the first event horizon telescope paper (L$1$ in [$1$]) says:




The appearance of M87* has been modeled successfully using GRMHD [general-relativistic magnetohydrodynamics] simulations, which describe a turbulent, hot, magnetized disk orbiting a Kerr black hole. They naturally produce a powerful jet and can explain the broadband spectral energy distribution observed in LLAGNs. At a wavelength of 1.3mm, and as observed here, the simulations also predict a shadow and an asymmetric emission ring.




Page 6 says:




...adopting an inclination of $17^circ$ between the approaching jet and the line of sight..., the west orientation of the jet, and a corotating disk model, matter in the bottom part of the image is moving toward the observer (clockwise rotation as seen from Earth). This is consistent with the rotation of the ionized gas on scales of 20 pc [20 parsecs, roughly 60 light-years], i.e., 7000 $r_g$ ["where $r_gequiv GM/c^2$ is the characteristic lengthscale of a black hole"]... and with the inferred sense of rotation from VLBI observations at 7 mm...




These excerpts say that when using black-hole parameters consistent with other observations, general relativity can predict the features of the images observed by the EHT. These features, including the reduced brightness in the center and the asymmetry of the brightness of the ring, with an orientation consistent with the observed jet, are hallmarks of a rapidly spinning black hole. In this sense, the images from the Event Horizon Telescope (EHT) do confirm general relativity.



The comparisons between general relativity's predictions and the observed images are described in detail in the fifth event horizon telescope paper (L$5$ in [$1$]), and some of them have already been reviewed on Physics SE:



  • Why isn't the circumferential light around the M87 black hole's event horizon symmetric?


References:



[$1$] https://iopscience.iop.org/issue/2041-8205/875/1, Table of contents of The Astrophysical Journal Letters, volume 875, number 1 (2019 April 10), with six downloadable articles (L$1$ thorugh L$6$)



[$2$] https://www.nasa.gov/feature/goddard/2017/messier-87



[$3$] "Black Hole-Powered Jet of Electrons and Sub-Atomic Particles Streams From Center of Galaxy M87," http://hubblesite.org/image/968/news_release/2000-20



[$4$] "Formation of the radio jet in M87 at 100 Schwarzschild radii from the central black hole," Nature 401, 891-892 (1999), https://www.nature.com/articles/44780



[$5$] "A precise measurement of the magnetic field in the corona of the black hole binary V404 Cygni," Science 358: 1299-1302 (2017), https://science.sciencemag.org/content/358/6368/1299



[$6$] "Fluorescent iron lines as a probe of astrophysical black hole systems," https://arxiv.org/abs/astro-ph/0212065



[$7$] Broderick et al, "The Event Horizon of M87," https://arxiv.org/abs/1503.03873






share|cite|improve this answer











$endgroup$




















    12












    $begingroup$

    If you google "m87 and general relativity" you get a list and videos on confirmation.



    This is an exaggerated response to an interesting "photograph", because it looks just like what has been calculated using the theory of general relativity for black holes.



    General relativity has been confirmed by many cosmological observations, including the calculations for the GPS signal and black holes were proposed within the framework of General relativity by Karl Schwarzschild . It is very interesting that the image developed exactly in the topology predicted by the GR equations, but the validation of GR did not really depend on this. (If a funny topology not predicted had been seen it would actually be more interesting because it would have to be modeled by something more complicated than a Kerr black hole., and maybe a modification to GR might have been proposed) .



    So the image is consistent with the expectation of a Kerr black hole, and in this sense it validates General Relativity.






    share|cite|improve this answer









    $endgroup$












    • $begingroup$
      Anna thank you . But if say so , you mean GR is what final theory of gravity or is it simply the best of we have explaining everything that come ahead. But I have very acute problem knowing why didn't he tell us about why exactly space curves
      $endgroup$
      – Liquid
      2 days ago






    • 8




      $begingroup$
      GR is the theory that fits our observations up to now. There are theorists trying to propose modification to GR. Well, in physics we cannot answer "Why" questions, but "how" from certain postulates using some equations we can fit observations. The "why these postulates and equations" belongs to metaphysics, not physics. Einstein was a physicist.
      $endgroup$
      – anna v
      2 days ago










    • $begingroup$
      @annav: I think the kind of "why" Liquid was talking about is less a matter of metaphysics and more in the sense of "why do these two pieces of iron seem to attract one another?"
      $endgroup$
      – Nicol Bolas
      2 days ago










    • $begingroup$
      There is no "final theory" in science. All of them are "the best we have".
      $endgroup$
      – nasu
      2 days ago










    • $begingroup$
      @NicolBolas yes . I mean it
      $endgroup$
      – Liquid
      yesterday


















    9












    $begingroup$

    There are two main aspects to the observations that are consistent with the predictions of GR and where the measurements stood a realistic chance of falsifying GR (and that is all you can do - design experiments capable of falsifying theories).



    The first is that the bright photon ring radius is to within about 10% of the prediction of GR for a black hole with a mass independently determined from the motions of stars in the central regions of M87.



    The second is that the black hole "shadow" is almost circular, to better than 10%. Again, this is predicted by GR in all but some extreme combinations of spin and spin-axis orientation. The maximum non circularity of the shadow caused by a Kerr black hole is 10% (see section 9.5 of the fifth Event Horizon Telescope M87 papers). Other explanations for the space-time distortion could have resulted in more oblate/prolate shadows.






    share|cite|improve this answer











    $endgroup$




















      6












      $begingroup$

      As some other answers say, the question "does this confirm General Relativity" gets it the wrong way round.



      GR is our current best understanding of how mass distorts spacetime, giving rise to many phenomenae we see around us. GR explains these as a result of distorted spacetime. It's incredibly accurate and has been tested in many, many ways.



      Without it, everyday things like satnav/GPS couldn't work, so it's also directly relevant to daily life. (GPS is so sensitive to timing that its calculations have to allow for the effects of General and Special Relativity)



      But because it's only our current best understanding, it's also likely and expected that GR will be eventually found to be "wrong enough on some things" that it will be superseded by some better theory in future. This happened to Newton's Laws of Motion - they are good enough for many parts of daily life, but we now know they aren't accurate in a lot of situations that exist, in which Special and General Relativity are more accurate.



      Why tests are important



      Because GR is our current "best theory", and relied on a lot, it is useful to know how accurately it matches experimental results, for at least 6 reasons:



      • to find (or double check) where we can rely on it, and if there are any places we can't rely on it. (This is especially valuable if the test was of of a kind that hasn't been done before, or if the test acts as a double-check on some existing test that hasn't been reproduced before).

      • to find where it seems to predict something which doesn't match what we see - this might suggest areas where we can look for "new physics" or unknown knowledge about the universe we are in, or alternatively, it can suggest how to improve in the care needed by experimenters and theorists, to avoid misinterpretations

      • to tell us if our experimental technique is sound, and whether a particular technique is capable of being used, or if it's working properly

      • For general education about science (in some high profile cases)

      • to explain to governments that their funding is useful and produces valuable knowledge. Funding is needed but must compete with other national needs, so showing results is useful.

      • Some tests require developing whole new methods, algorithms, computer technology, and analytic techniques, which then have wider benefits. For example, the Large Hadron Collider required development of entire data storage and distributed networking methods which have been used in cloud computing and the wider computing world.

      The Black Hole photograph's importance



      In that sense, the imaging of a black hole is important. That isn't because it 'proves" GR is correct (it can't), but because of these same 6 reasons:




      • it confirms that GR is reliable enough that what it predicts a Black Hole should look like, if we could ever photograph one, really does seem to match what we see when we do actually photograph one from billions of light years away. Even with the naked eye, we can see the accretion disk, the shadow, the black centre, and the effects of rotation and temperature on the bright areas. Probably when the image is analysed very closely, it might also confirm that we see many other things that match what GR said we should expect, which aren't easy to see with the naked eye. The photograph could have looked very different from the prediction, but it didn't.
        It is also the first test of its kind.


      • it may contain extra data that when studied, lets us improve current theories about the universe (and the lives of objects within it), or helps us to narrow down what theories and parameters can be "correct"


      • it tells us that our technique of ultra long range photography using this new technique is sound, so we can move forward and improve it, or use it to view other objects directly.

      • it is dramatic, so many people will read it and learn more about the universe.
        So it can be educational


      • it shows that the funding aim was successfully achieved when the project was funded.


      • it required whole new algorithms and techniques to be developed, which may be beneficial to society or in other future projects, and which can themselves be built upon and expanded.
        Nothing like it, on this scale, had been tried before.

      Comment



      It's only in that sense that this experiment and the Black Hole photo "confirms" anything. But it can still only be one more piece of evidence that so far, GR isn't letting us down.



      It doesn't change the fact that ultimately we do expect that some limit to GR's accuracy will probably be found, or we will encounter a new situation where GR doesn't apply very well. When that happens, we hope that a better theory will arise that matches GR in the areas GR works, and also matches those new findings in the areas GR doesn't work. So far, this isn't a test that shows a limit has been found, though.



      So it's win-win really, either we prove our existing theories work in an even more extreme or novel case, or we get a hint of an unexpected difference which will help us to understand things better (either improving our tests to remove a mistake, or improving our theory if the test was correct).






      share|cite|improve this answer











      $endgroup$













        Your Answer








        StackExchange.ready(function()
        var channelOptions =
        tags: "".split(" "),
        id: "151"
        ;
        initTagRenderer("".split(" "), "".split(" "), channelOptions);

        StackExchange.using("externalEditor", function()
        // Have to fire editor after snippets, if snippets enabled
        if (StackExchange.settings.snippets.snippetsEnabled)
        StackExchange.using("snippets", function()
        createEditor();
        );

        else
        createEditor();

        );

        function createEditor()
        StackExchange.prepareEditor(
        heartbeatType: 'answer',
        autoActivateHeartbeat: false,
        convertImagesToLinks: false,
        noModals: true,
        showLowRepImageUploadWarning: true,
        reputationToPostImages: null,
        bindNavPrevention: true,
        postfix: "",
        imageUploader:
        brandingHtml: "Powered by u003ca class="icon-imgur-white" href="https://imgur.com/"u003eu003c/au003e",
        contentPolicyHtml: "User contributions licensed under u003ca href="https://creativecommons.org/licenses/by-sa/3.0/"u003ecc by-sa 3.0 with attribution requiredu003c/au003e u003ca href="https://stackoverflow.com/legal/content-policy"u003e(content policy)u003c/au003e",
        allowUrls: true
        ,
        noCode: true, onDemand: true,
        discardSelector: ".discard-answer"
        ,immediatelyShowMarkdownHelp:true
        );



        );













        draft saved

        draft discarded


















        StackExchange.ready(
        function ()
        StackExchange.openid.initPostLogin('.new-post-login', 'https%3a%2f%2fphysics.stackexchange.com%2fquestions%2f472323%2fdid-the-new-image-of-black-hole-confirm-the-general-theory-of-relativity%23new-answer', 'question_page');

        );

        Post as a guest















        Required, but never shown

























        5 Answers
        5






        active

        oldest

        votes








        5 Answers
        5






        active

        oldest

        votes









        active

        oldest

        votes






        active

        oldest

        votes









        49












        $begingroup$

        I think it's fair to say that the EHT image definitely is consistent with GR, and so GR continues to agree with experimental data so far. The leading paper in the 10th April 2019 issue of Astrophysical Journal letters says (first sentence of the 'Discussion' section):




        A number of elements reinforce the robustness of our image and the conclusion that it is consistent with the shadow of a black hole as predicted by GR.




        I'm unhappy about the notion that this 'confirms' GR: it would be more correct to say that GR has not been shown to be wrong by this observation: nothing can definitively confirm a theory, which can only be shown to agree with experimental data so far.



        This depends of course on the definition of 'confirm': above I am taking it to mean 'shown to be correct' which I think is the everyday usage and the one implied in your question, and it's that meaning I object to. In particular it is clearly not the case that this shows 'Einstein was right': it shows that GR agrees with experiment (extremely well!) so far, and this and LIGO both show (or are showing) that GR agrees with experiment in regions where the gravitational field is strong.



        (Note that, when used informally by scientists, 'confirm' very often means exactly 'shown to agree with experiment so far' and in that sense GR has been confirmed (again) by this observation. I'm assuming that this is not the meaning you meant however.)




        At least one other answer to this question is excellent and very much worth reading in addition to this.






        share|cite|improve this answer











        $endgroup$








        • 10




          $begingroup$
          It "confirms" it in the same way that the observation of gravitational lensing during the 1919 solar eclipse confirmed it.
          $endgroup$
          – Barmar
          2 days ago






        • 11




          $begingroup$
          @Barmar which is to say, it didn't disprove it. That's the best a theory can hope. "Confirm" is such an imprecise word that makes non-scientists get completely the wrong idea about the role of evidence in science.
          $endgroup$
          – Roman Starkov
          2 days ago






        • 1




          $begingroup$
          Yet it was good enough to make Einstein a household name. I think what makes some things more confirming is that the prediction they agree with is something quite unexpected according to previous theories. I'm not sure if the same thing could be said for the EHT pictures, because of all the evidence we've already found in the past century.
          $endgroup$
          – Barmar
          2 days ago






        • 2




          $begingroup$
          " it would be more correct to say that GR has not been shown to be wrong by this observation: nothing can definitively confirm a theory," Sorry, but you've got the terminology wrong. A theory is tested by comparing its predictions against experimental data. If the data does not agree with the theory, the theory is disproved (at least, provisionally). If the data does agree, the theory is confirmed (but not proven). You are confusing proof with confirmation.
          $endgroup$
          – WhatRoughBeast
          yesterday






        • 1




          $begingroup$
          @WhatRoughBeast: Isn't theory/hypothesis confusing the general public enough already? Is it really, absolutely necessary to invent another bit of jargon on top of that?
          $endgroup$
          – Kevin
          yesterday
















        49












        $begingroup$

        I think it's fair to say that the EHT image definitely is consistent with GR, and so GR continues to agree with experimental data so far. The leading paper in the 10th April 2019 issue of Astrophysical Journal letters says (first sentence of the 'Discussion' section):




        A number of elements reinforce the robustness of our image and the conclusion that it is consistent with the shadow of a black hole as predicted by GR.




        I'm unhappy about the notion that this 'confirms' GR: it would be more correct to say that GR has not been shown to be wrong by this observation: nothing can definitively confirm a theory, which can only be shown to agree with experimental data so far.



        This depends of course on the definition of 'confirm': above I am taking it to mean 'shown to be correct' which I think is the everyday usage and the one implied in your question, and it's that meaning I object to. In particular it is clearly not the case that this shows 'Einstein was right': it shows that GR agrees with experiment (extremely well!) so far, and this and LIGO both show (or are showing) that GR agrees with experiment in regions where the gravitational field is strong.



        (Note that, when used informally by scientists, 'confirm' very often means exactly 'shown to agree with experiment so far' and in that sense GR has been confirmed (again) by this observation. I'm assuming that this is not the meaning you meant however.)




        At least one other answer to this question is excellent and very much worth reading in addition to this.






        share|cite|improve this answer











        $endgroup$








        • 10




          $begingroup$
          It "confirms" it in the same way that the observation of gravitational lensing during the 1919 solar eclipse confirmed it.
          $endgroup$
          – Barmar
          2 days ago






        • 11




          $begingroup$
          @Barmar which is to say, it didn't disprove it. That's the best a theory can hope. "Confirm" is such an imprecise word that makes non-scientists get completely the wrong idea about the role of evidence in science.
          $endgroup$
          – Roman Starkov
          2 days ago






        • 1




          $begingroup$
          Yet it was good enough to make Einstein a household name. I think what makes some things more confirming is that the prediction they agree with is something quite unexpected according to previous theories. I'm not sure if the same thing could be said for the EHT pictures, because of all the evidence we've already found in the past century.
          $endgroup$
          – Barmar
          2 days ago






        • 2




          $begingroup$
          " it would be more correct to say that GR has not been shown to be wrong by this observation: nothing can definitively confirm a theory," Sorry, but you've got the terminology wrong. A theory is tested by comparing its predictions against experimental data. If the data does not agree with the theory, the theory is disproved (at least, provisionally). If the data does agree, the theory is confirmed (but not proven). You are confusing proof with confirmation.
          $endgroup$
          – WhatRoughBeast
          yesterday






        • 1




          $begingroup$
          @WhatRoughBeast: Isn't theory/hypothesis confusing the general public enough already? Is it really, absolutely necessary to invent another bit of jargon on top of that?
          $endgroup$
          – Kevin
          yesterday














        49












        49








        49





        $begingroup$

        I think it's fair to say that the EHT image definitely is consistent with GR, and so GR continues to agree with experimental data so far. The leading paper in the 10th April 2019 issue of Astrophysical Journal letters says (first sentence of the 'Discussion' section):




        A number of elements reinforce the robustness of our image and the conclusion that it is consistent with the shadow of a black hole as predicted by GR.




        I'm unhappy about the notion that this 'confirms' GR: it would be more correct to say that GR has not been shown to be wrong by this observation: nothing can definitively confirm a theory, which can only be shown to agree with experimental data so far.



        This depends of course on the definition of 'confirm': above I am taking it to mean 'shown to be correct' which I think is the everyday usage and the one implied in your question, and it's that meaning I object to. In particular it is clearly not the case that this shows 'Einstein was right': it shows that GR agrees with experiment (extremely well!) so far, and this and LIGO both show (or are showing) that GR agrees with experiment in regions where the gravitational field is strong.



        (Note that, when used informally by scientists, 'confirm' very often means exactly 'shown to agree with experiment so far' and in that sense GR has been confirmed (again) by this observation. I'm assuming that this is not the meaning you meant however.)




        At least one other answer to this question is excellent and very much worth reading in addition to this.






        share|cite|improve this answer











        $endgroup$



        I think it's fair to say that the EHT image definitely is consistent with GR, and so GR continues to agree with experimental data so far. The leading paper in the 10th April 2019 issue of Astrophysical Journal letters says (first sentence of the 'Discussion' section):




        A number of elements reinforce the robustness of our image and the conclusion that it is consistent with the shadow of a black hole as predicted by GR.




        I'm unhappy about the notion that this 'confirms' GR: it would be more correct to say that GR has not been shown to be wrong by this observation: nothing can definitively confirm a theory, which can only be shown to agree with experimental data so far.



        This depends of course on the definition of 'confirm': above I am taking it to mean 'shown to be correct' which I think is the everyday usage and the one implied in your question, and it's that meaning I object to. In particular it is clearly not the case that this shows 'Einstein was right': it shows that GR agrees with experiment (extremely well!) so far, and this and LIGO both show (or are showing) that GR agrees with experiment in regions where the gravitational field is strong.



        (Note that, when used informally by scientists, 'confirm' very often means exactly 'shown to agree with experiment so far' and in that sense GR has been confirmed (again) by this observation. I'm assuming that this is not the meaning you meant however.)




        At least one other answer to this question is excellent and very much worth reading in addition to this.







        share|cite|improve this answer














        share|cite|improve this answer



        share|cite|improve this answer








        edited 11 hours ago

























        answered 2 days ago









        tfbtfb

        16k43452




        16k43452







        • 10




          $begingroup$
          It "confirms" it in the same way that the observation of gravitational lensing during the 1919 solar eclipse confirmed it.
          $endgroup$
          – Barmar
          2 days ago






        • 11




          $begingroup$
          @Barmar which is to say, it didn't disprove it. That's the best a theory can hope. "Confirm" is such an imprecise word that makes non-scientists get completely the wrong idea about the role of evidence in science.
          $endgroup$
          – Roman Starkov
          2 days ago






        • 1




          $begingroup$
          Yet it was good enough to make Einstein a household name. I think what makes some things more confirming is that the prediction they agree with is something quite unexpected according to previous theories. I'm not sure if the same thing could be said for the EHT pictures, because of all the evidence we've already found in the past century.
          $endgroup$
          – Barmar
          2 days ago






        • 2




          $begingroup$
          " it would be more correct to say that GR has not been shown to be wrong by this observation: nothing can definitively confirm a theory," Sorry, but you've got the terminology wrong. A theory is tested by comparing its predictions against experimental data. If the data does not agree with the theory, the theory is disproved (at least, provisionally). If the data does agree, the theory is confirmed (but not proven). You are confusing proof with confirmation.
          $endgroup$
          – WhatRoughBeast
          yesterday






        • 1




          $begingroup$
          @WhatRoughBeast: Isn't theory/hypothesis confusing the general public enough already? Is it really, absolutely necessary to invent another bit of jargon on top of that?
          $endgroup$
          – Kevin
          yesterday













        • 10




          $begingroup$
          It "confirms" it in the same way that the observation of gravitational lensing during the 1919 solar eclipse confirmed it.
          $endgroup$
          – Barmar
          2 days ago






        • 11




          $begingroup$
          @Barmar which is to say, it didn't disprove it. That's the best a theory can hope. "Confirm" is such an imprecise word that makes non-scientists get completely the wrong idea about the role of evidence in science.
          $endgroup$
          – Roman Starkov
          2 days ago






        • 1




          $begingroup$
          Yet it was good enough to make Einstein a household name. I think what makes some things more confirming is that the prediction they agree with is something quite unexpected according to previous theories. I'm not sure if the same thing could be said for the EHT pictures, because of all the evidence we've already found in the past century.
          $endgroup$
          – Barmar
          2 days ago






        • 2




          $begingroup$
          " it would be more correct to say that GR has not been shown to be wrong by this observation: nothing can definitively confirm a theory," Sorry, but you've got the terminology wrong. A theory is tested by comparing its predictions against experimental data. If the data does not agree with the theory, the theory is disproved (at least, provisionally). If the data does agree, the theory is confirmed (but not proven). You are confusing proof with confirmation.
          $endgroup$
          – WhatRoughBeast
          yesterday






        • 1




          $begingroup$
          @WhatRoughBeast: Isn't theory/hypothesis confusing the general public enough already? Is it really, absolutely necessary to invent another bit of jargon on top of that?
          $endgroup$
          – Kevin
          yesterday








        10




        10




        $begingroup$
        It "confirms" it in the same way that the observation of gravitational lensing during the 1919 solar eclipse confirmed it.
        $endgroup$
        – Barmar
        2 days ago




        $begingroup$
        It "confirms" it in the same way that the observation of gravitational lensing during the 1919 solar eclipse confirmed it.
        $endgroup$
        – Barmar
        2 days ago




        11




        11




        $begingroup$
        @Barmar which is to say, it didn't disprove it. That's the best a theory can hope. "Confirm" is such an imprecise word that makes non-scientists get completely the wrong idea about the role of evidence in science.
        $endgroup$
        – Roman Starkov
        2 days ago




        $begingroup$
        @Barmar which is to say, it didn't disprove it. That's the best a theory can hope. "Confirm" is such an imprecise word that makes non-scientists get completely the wrong idea about the role of evidence in science.
        $endgroup$
        – Roman Starkov
        2 days ago




        1




        1




        $begingroup$
        Yet it was good enough to make Einstein a household name. I think what makes some things more confirming is that the prediction they agree with is something quite unexpected according to previous theories. I'm not sure if the same thing could be said for the EHT pictures, because of all the evidence we've already found in the past century.
        $endgroup$
        – Barmar
        2 days ago




        $begingroup$
        Yet it was good enough to make Einstein a household name. I think what makes some things more confirming is that the prediction they agree with is something quite unexpected according to previous theories. I'm not sure if the same thing could be said for the EHT pictures, because of all the evidence we've already found in the past century.
        $endgroup$
        – Barmar
        2 days ago




        2




        2




        $begingroup$
        " it would be more correct to say that GR has not been shown to be wrong by this observation: nothing can definitively confirm a theory," Sorry, but you've got the terminology wrong. A theory is tested by comparing its predictions against experimental data. If the data does not agree with the theory, the theory is disproved (at least, provisionally). If the data does agree, the theory is confirmed (but not proven). You are confusing proof with confirmation.
        $endgroup$
        – WhatRoughBeast
        yesterday




        $begingroup$
        " it would be more correct to say that GR has not been shown to be wrong by this observation: nothing can definitively confirm a theory," Sorry, but you've got the terminology wrong. A theory is tested by comparing its predictions against experimental data. If the data does not agree with the theory, the theory is disproved (at least, provisionally). If the data does agree, the theory is confirmed (but not proven). You are confusing proof with confirmation.
        $endgroup$
        – WhatRoughBeast
        yesterday




        1




        1




        $begingroup$
        @WhatRoughBeast: Isn't theory/hypothesis confusing the general public enough already? Is it really, absolutely necessary to invent another bit of jargon on top of that?
        $endgroup$
        – Kevin
        yesterday





        $begingroup$
        @WhatRoughBeast: Isn't theory/hypothesis confusing the general public enough already? Is it really, absolutely necessary to invent another bit of jargon on top of that?
        $endgroup$
        – Kevin
        yesterday












        28












        $begingroup$

        Answer: The images from the Event Horizon Telescope (EHT) are consistent with what general relativity predicts. So if that's what the OP means by "confirm general relativity", then the answer is yes.



        To appreciate the significance of the EHT images, we need to remember how science works. Theories are inspired by observations, but theories are not deduced from observations; we certainly cannot deduce general relativity from a single image. It's the other way around: theories make predictions. If the predictions consistently match what we observe, then we say that the theory works. Of course, contriving a theory that makes one prediction that matches one observation is trivial. Finding a theory that makes many predictions that match many observations is more challenging. General relativity is such a theory, and its agreement with this new observation — the images from the Event Horizon Telescope — is a nice addition to the large portfolio of general relativity's confirmed predictions.



        Even though the black hole is enormous, it is so far away that the diameter of the imaged ring spans less than $20$ billionths of a degree ($<20times 10^-9$ degree) in the sky, so pristine resolution cannot be expected; the fact that they were able to resolve it at all is remarkable. Still, the image shows some general features that are consistent with what is expected from the light-bending effects associated with a rapidly spinning black hole in general relativity — not just any rapidly spinning black hole, but one whose size, mass, spin, and orientation are all consistent with other observations associated with that same black hole in the core of the galaxy M87.



        A few of these observations are reviewed below, followed by comments on how the EHT images compare to predictions from general relativity.




        Other observations: The jet



        One of the most prominent associated observations is the jet emanating from the galaxy's core, shown here in images from the Hubble Space Telescope [$2$]:



        enter image description here



        To give a feeling for the scale of this picture, this is what hubblesite.org [$3$] says about the image:




        At a distance of 50 million light-years, M87 is too distant for Hubble to discern individual stars. The dozens of star-like points swarming about M87 are, instead, themselves clusters of hundreds of thousands of stars each.




        Here's another view of the jet, with scale-bars:



        enter image description here



        This image (from figure 2 in [$4$]) was made in 1999 using VLBI observations at a wavelength of 7 millimeters. The white dot marked $6r_S$ represents a circle with a diameter of $6$ times the alleged Schwarzschild radius. The scale bar marked "$1$ kpc" represents one kiloparsec, which is roughly 3000 light-years.



        According to general relativity, a rapidly spinning black hole with an accretion disk can generate intense magnetic fields (but see [$5$]) that funnel material from the accreting plasma into a jet emanating along the black hole's axis of rotation. The fact that the observed jet is so straight over a distance of thousands of light-years implies that it must be produced by an engine that maintains a very consistent orientation for a time span of at least thousands of years, as a supermassive black hole is expected to do.




        Other observations: The accretion disk



        According to [$6$]:




        HST [Hubble Space Telescope] imaged a disk of ionized gas, with a radius of $sim$ 50 pc [50 parsecs, roughly 150 light-years] centered on the galactic core... The high resolution of HST allowed the spectrum [which is sensitive to the Doppler effect] of this ionized gas to be measured as a function of position across the gas disk, thereby allowing the kinematics of the disk to be determined... It was found that the velocity profile of the central 20 pc of the gas disk possessed a Keplerian profile (i.e., $v propto r^-1/2$) as expected if the gas was orbiting in the gravitational potential of a point-like mass... The only known and long-lived object to possess such a large mass in a small region of space, and be as under-luminous as observed, is a SMBH [Super-Massive Black Hole].




        In other words, these observations showed evidence for gas disk with the velocity profile that would be expected if it were orbiting a supermassive black hole. Note that the measured gas velocities on opposite sides of the central body differ from each other by roughly 1000 kilometers per second.




        Other observations: The absense of strong surface emission



        According to a report [$7$] published in 2015:




        Observations at millimeter wavelengths with the Event Horizon Telescope have localized the emission from the base of this jet [shown above] to angular scales comparable to the putative black hole horizon. The jet might be powered directly by an accretion disk or by electromagnetic extraction of the rotational energy of the black hole. However, even the latter mechanism requires a confining thick accretion disk to maintain the required magnetic flux near the black hole. Therefore, regardless of the jet mechanism, the observed jet power in M87 implies a certain minimum mass accretion rate. If the central compact object in M87 were not a black hole but had a surface, this accretion would result in considerable thermal near-infrared and optical emission from the surface. Current flux limits on the nucleus of M87 strongly constrain any such surface emission. This rules out the presence of a surface and thereby provides indirect evidence for an event horizon.




        Regarding why the event horizon of a black hole is expected to be so dim even though the intense fields generate a powerful jet, see these Physics SE posts:



        • How can Quasars emit anything if they're black holes?


        • How can cosmic jets exist?



        Comparing the EHT images to predictions



        Page 5 in the first event horizon telescope paper (L$1$ in [$1$]) says:




        The appearance of M87* has been modeled successfully using GRMHD [general-relativistic magnetohydrodynamics] simulations, which describe a turbulent, hot, magnetized disk orbiting a Kerr black hole. They naturally produce a powerful jet and can explain the broadband spectral energy distribution observed in LLAGNs. At a wavelength of 1.3mm, and as observed here, the simulations also predict a shadow and an asymmetric emission ring.




        Page 6 says:




        ...adopting an inclination of $17^circ$ between the approaching jet and the line of sight..., the west orientation of the jet, and a corotating disk model, matter in the bottom part of the image is moving toward the observer (clockwise rotation as seen from Earth). This is consistent with the rotation of the ionized gas on scales of 20 pc [20 parsecs, roughly 60 light-years], i.e., 7000 $r_g$ ["where $r_gequiv GM/c^2$ is the characteristic lengthscale of a black hole"]... and with the inferred sense of rotation from VLBI observations at 7 mm...




        These excerpts say that when using black-hole parameters consistent with other observations, general relativity can predict the features of the images observed by the EHT. These features, including the reduced brightness in the center and the asymmetry of the brightness of the ring, with an orientation consistent with the observed jet, are hallmarks of a rapidly spinning black hole. In this sense, the images from the Event Horizon Telescope (EHT) do confirm general relativity.



        The comparisons between general relativity's predictions and the observed images are described in detail in the fifth event horizon telescope paper (L$5$ in [$1$]), and some of them have already been reviewed on Physics SE:



        • Why isn't the circumferential light around the M87 black hole's event horizon symmetric?


        References:



        [$1$] https://iopscience.iop.org/issue/2041-8205/875/1, Table of contents of The Astrophysical Journal Letters, volume 875, number 1 (2019 April 10), with six downloadable articles (L$1$ thorugh L$6$)



        [$2$] https://www.nasa.gov/feature/goddard/2017/messier-87



        [$3$] "Black Hole-Powered Jet of Electrons and Sub-Atomic Particles Streams From Center of Galaxy M87," http://hubblesite.org/image/968/news_release/2000-20



        [$4$] "Formation of the radio jet in M87 at 100 Schwarzschild radii from the central black hole," Nature 401, 891-892 (1999), https://www.nature.com/articles/44780



        [$5$] "A precise measurement of the magnetic field in the corona of the black hole binary V404 Cygni," Science 358: 1299-1302 (2017), https://science.sciencemag.org/content/358/6368/1299



        [$6$] "Fluorescent iron lines as a probe of astrophysical black hole systems," https://arxiv.org/abs/astro-ph/0212065



        [$7$] Broderick et al, "The Event Horizon of M87," https://arxiv.org/abs/1503.03873






        share|cite|improve this answer











        $endgroup$

















          28












          $begingroup$

          Answer: The images from the Event Horizon Telescope (EHT) are consistent with what general relativity predicts. So if that's what the OP means by "confirm general relativity", then the answer is yes.



          To appreciate the significance of the EHT images, we need to remember how science works. Theories are inspired by observations, but theories are not deduced from observations; we certainly cannot deduce general relativity from a single image. It's the other way around: theories make predictions. If the predictions consistently match what we observe, then we say that the theory works. Of course, contriving a theory that makes one prediction that matches one observation is trivial. Finding a theory that makes many predictions that match many observations is more challenging. General relativity is such a theory, and its agreement with this new observation — the images from the Event Horizon Telescope — is a nice addition to the large portfolio of general relativity's confirmed predictions.



          Even though the black hole is enormous, it is so far away that the diameter of the imaged ring spans less than $20$ billionths of a degree ($<20times 10^-9$ degree) in the sky, so pristine resolution cannot be expected; the fact that they were able to resolve it at all is remarkable. Still, the image shows some general features that are consistent with what is expected from the light-bending effects associated with a rapidly spinning black hole in general relativity — not just any rapidly spinning black hole, but one whose size, mass, spin, and orientation are all consistent with other observations associated with that same black hole in the core of the galaxy M87.



          A few of these observations are reviewed below, followed by comments on how the EHT images compare to predictions from general relativity.




          Other observations: The jet



          One of the most prominent associated observations is the jet emanating from the galaxy's core, shown here in images from the Hubble Space Telescope [$2$]:



          enter image description here



          To give a feeling for the scale of this picture, this is what hubblesite.org [$3$] says about the image:




          At a distance of 50 million light-years, M87 is too distant for Hubble to discern individual stars. The dozens of star-like points swarming about M87 are, instead, themselves clusters of hundreds of thousands of stars each.




          Here's another view of the jet, with scale-bars:



          enter image description here



          This image (from figure 2 in [$4$]) was made in 1999 using VLBI observations at a wavelength of 7 millimeters. The white dot marked $6r_S$ represents a circle with a diameter of $6$ times the alleged Schwarzschild radius. The scale bar marked "$1$ kpc" represents one kiloparsec, which is roughly 3000 light-years.



          According to general relativity, a rapidly spinning black hole with an accretion disk can generate intense magnetic fields (but see [$5$]) that funnel material from the accreting plasma into a jet emanating along the black hole's axis of rotation. The fact that the observed jet is so straight over a distance of thousands of light-years implies that it must be produced by an engine that maintains a very consistent orientation for a time span of at least thousands of years, as a supermassive black hole is expected to do.




          Other observations: The accretion disk



          According to [$6$]:




          HST [Hubble Space Telescope] imaged a disk of ionized gas, with a radius of $sim$ 50 pc [50 parsecs, roughly 150 light-years] centered on the galactic core... The high resolution of HST allowed the spectrum [which is sensitive to the Doppler effect] of this ionized gas to be measured as a function of position across the gas disk, thereby allowing the kinematics of the disk to be determined... It was found that the velocity profile of the central 20 pc of the gas disk possessed a Keplerian profile (i.e., $v propto r^-1/2$) as expected if the gas was orbiting in the gravitational potential of a point-like mass... The only known and long-lived object to possess such a large mass in a small region of space, and be as under-luminous as observed, is a SMBH [Super-Massive Black Hole].




          In other words, these observations showed evidence for gas disk with the velocity profile that would be expected if it were orbiting a supermassive black hole. Note that the measured gas velocities on opposite sides of the central body differ from each other by roughly 1000 kilometers per second.




          Other observations: The absense of strong surface emission



          According to a report [$7$] published in 2015:




          Observations at millimeter wavelengths with the Event Horizon Telescope have localized the emission from the base of this jet [shown above] to angular scales comparable to the putative black hole horizon. The jet might be powered directly by an accretion disk or by electromagnetic extraction of the rotational energy of the black hole. However, even the latter mechanism requires a confining thick accretion disk to maintain the required magnetic flux near the black hole. Therefore, regardless of the jet mechanism, the observed jet power in M87 implies a certain minimum mass accretion rate. If the central compact object in M87 were not a black hole but had a surface, this accretion would result in considerable thermal near-infrared and optical emission from the surface. Current flux limits on the nucleus of M87 strongly constrain any such surface emission. This rules out the presence of a surface and thereby provides indirect evidence for an event horizon.




          Regarding why the event horizon of a black hole is expected to be so dim even though the intense fields generate a powerful jet, see these Physics SE posts:



          • How can Quasars emit anything if they're black holes?


          • How can cosmic jets exist?



          Comparing the EHT images to predictions



          Page 5 in the first event horizon telescope paper (L$1$ in [$1$]) says:




          The appearance of M87* has been modeled successfully using GRMHD [general-relativistic magnetohydrodynamics] simulations, which describe a turbulent, hot, magnetized disk orbiting a Kerr black hole. They naturally produce a powerful jet and can explain the broadband spectral energy distribution observed in LLAGNs. At a wavelength of 1.3mm, and as observed here, the simulations also predict a shadow and an asymmetric emission ring.




          Page 6 says:




          ...adopting an inclination of $17^circ$ between the approaching jet and the line of sight..., the west orientation of the jet, and a corotating disk model, matter in the bottom part of the image is moving toward the observer (clockwise rotation as seen from Earth). This is consistent with the rotation of the ionized gas on scales of 20 pc [20 parsecs, roughly 60 light-years], i.e., 7000 $r_g$ ["where $r_gequiv GM/c^2$ is the characteristic lengthscale of a black hole"]... and with the inferred sense of rotation from VLBI observations at 7 mm...




          These excerpts say that when using black-hole parameters consistent with other observations, general relativity can predict the features of the images observed by the EHT. These features, including the reduced brightness in the center and the asymmetry of the brightness of the ring, with an orientation consistent with the observed jet, are hallmarks of a rapidly spinning black hole. In this sense, the images from the Event Horizon Telescope (EHT) do confirm general relativity.



          The comparisons between general relativity's predictions and the observed images are described in detail in the fifth event horizon telescope paper (L$5$ in [$1$]), and some of them have already been reviewed on Physics SE:



          • Why isn't the circumferential light around the M87 black hole's event horizon symmetric?


          References:



          [$1$] https://iopscience.iop.org/issue/2041-8205/875/1, Table of contents of The Astrophysical Journal Letters, volume 875, number 1 (2019 April 10), with six downloadable articles (L$1$ thorugh L$6$)



          [$2$] https://www.nasa.gov/feature/goddard/2017/messier-87



          [$3$] "Black Hole-Powered Jet of Electrons and Sub-Atomic Particles Streams From Center of Galaxy M87," http://hubblesite.org/image/968/news_release/2000-20



          [$4$] "Formation of the radio jet in M87 at 100 Schwarzschild radii from the central black hole," Nature 401, 891-892 (1999), https://www.nature.com/articles/44780



          [$5$] "A precise measurement of the magnetic field in the corona of the black hole binary V404 Cygni," Science 358: 1299-1302 (2017), https://science.sciencemag.org/content/358/6368/1299



          [$6$] "Fluorescent iron lines as a probe of astrophysical black hole systems," https://arxiv.org/abs/astro-ph/0212065



          [$7$] Broderick et al, "The Event Horizon of M87," https://arxiv.org/abs/1503.03873






          share|cite|improve this answer











          $endgroup$















            28












            28








            28





            $begingroup$

            Answer: The images from the Event Horizon Telescope (EHT) are consistent with what general relativity predicts. So if that's what the OP means by "confirm general relativity", then the answer is yes.



            To appreciate the significance of the EHT images, we need to remember how science works. Theories are inspired by observations, but theories are not deduced from observations; we certainly cannot deduce general relativity from a single image. It's the other way around: theories make predictions. If the predictions consistently match what we observe, then we say that the theory works. Of course, contriving a theory that makes one prediction that matches one observation is trivial. Finding a theory that makes many predictions that match many observations is more challenging. General relativity is such a theory, and its agreement with this new observation — the images from the Event Horizon Telescope — is a nice addition to the large portfolio of general relativity's confirmed predictions.



            Even though the black hole is enormous, it is so far away that the diameter of the imaged ring spans less than $20$ billionths of a degree ($<20times 10^-9$ degree) in the sky, so pristine resolution cannot be expected; the fact that they were able to resolve it at all is remarkable. Still, the image shows some general features that are consistent with what is expected from the light-bending effects associated with a rapidly spinning black hole in general relativity — not just any rapidly spinning black hole, but one whose size, mass, spin, and orientation are all consistent with other observations associated with that same black hole in the core of the galaxy M87.



            A few of these observations are reviewed below, followed by comments on how the EHT images compare to predictions from general relativity.




            Other observations: The jet



            One of the most prominent associated observations is the jet emanating from the galaxy's core, shown here in images from the Hubble Space Telescope [$2$]:



            enter image description here



            To give a feeling for the scale of this picture, this is what hubblesite.org [$3$] says about the image:




            At a distance of 50 million light-years, M87 is too distant for Hubble to discern individual stars. The dozens of star-like points swarming about M87 are, instead, themselves clusters of hundreds of thousands of stars each.




            Here's another view of the jet, with scale-bars:



            enter image description here



            This image (from figure 2 in [$4$]) was made in 1999 using VLBI observations at a wavelength of 7 millimeters. The white dot marked $6r_S$ represents a circle with a diameter of $6$ times the alleged Schwarzschild radius. The scale bar marked "$1$ kpc" represents one kiloparsec, which is roughly 3000 light-years.



            According to general relativity, a rapidly spinning black hole with an accretion disk can generate intense magnetic fields (but see [$5$]) that funnel material from the accreting plasma into a jet emanating along the black hole's axis of rotation. The fact that the observed jet is so straight over a distance of thousands of light-years implies that it must be produced by an engine that maintains a very consistent orientation for a time span of at least thousands of years, as a supermassive black hole is expected to do.




            Other observations: The accretion disk



            According to [$6$]:




            HST [Hubble Space Telescope] imaged a disk of ionized gas, with a radius of $sim$ 50 pc [50 parsecs, roughly 150 light-years] centered on the galactic core... The high resolution of HST allowed the spectrum [which is sensitive to the Doppler effect] of this ionized gas to be measured as a function of position across the gas disk, thereby allowing the kinematics of the disk to be determined... It was found that the velocity profile of the central 20 pc of the gas disk possessed a Keplerian profile (i.e., $v propto r^-1/2$) as expected if the gas was orbiting in the gravitational potential of a point-like mass... The only known and long-lived object to possess such a large mass in a small region of space, and be as under-luminous as observed, is a SMBH [Super-Massive Black Hole].




            In other words, these observations showed evidence for gas disk with the velocity profile that would be expected if it were orbiting a supermassive black hole. Note that the measured gas velocities on opposite sides of the central body differ from each other by roughly 1000 kilometers per second.




            Other observations: The absense of strong surface emission



            According to a report [$7$] published in 2015:




            Observations at millimeter wavelengths with the Event Horizon Telescope have localized the emission from the base of this jet [shown above] to angular scales comparable to the putative black hole horizon. The jet might be powered directly by an accretion disk or by electromagnetic extraction of the rotational energy of the black hole. However, even the latter mechanism requires a confining thick accretion disk to maintain the required magnetic flux near the black hole. Therefore, regardless of the jet mechanism, the observed jet power in M87 implies a certain minimum mass accretion rate. If the central compact object in M87 were not a black hole but had a surface, this accretion would result in considerable thermal near-infrared and optical emission from the surface. Current flux limits on the nucleus of M87 strongly constrain any such surface emission. This rules out the presence of a surface and thereby provides indirect evidence for an event horizon.




            Regarding why the event horizon of a black hole is expected to be so dim even though the intense fields generate a powerful jet, see these Physics SE posts:



            • How can Quasars emit anything if they're black holes?


            • How can cosmic jets exist?



            Comparing the EHT images to predictions



            Page 5 in the first event horizon telescope paper (L$1$ in [$1$]) says:




            The appearance of M87* has been modeled successfully using GRMHD [general-relativistic magnetohydrodynamics] simulations, which describe a turbulent, hot, magnetized disk orbiting a Kerr black hole. They naturally produce a powerful jet and can explain the broadband spectral energy distribution observed in LLAGNs. At a wavelength of 1.3mm, and as observed here, the simulations also predict a shadow and an asymmetric emission ring.




            Page 6 says:




            ...adopting an inclination of $17^circ$ between the approaching jet and the line of sight..., the west orientation of the jet, and a corotating disk model, matter in the bottom part of the image is moving toward the observer (clockwise rotation as seen from Earth). This is consistent with the rotation of the ionized gas on scales of 20 pc [20 parsecs, roughly 60 light-years], i.e., 7000 $r_g$ ["where $r_gequiv GM/c^2$ is the characteristic lengthscale of a black hole"]... and with the inferred sense of rotation from VLBI observations at 7 mm...




            These excerpts say that when using black-hole parameters consistent with other observations, general relativity can predict the features of the images observed by the EHT. These features, including the reduced brightness in the center and the asymmetry of the brightness of the ring, with an orientation consistent with the observed jet, are hallmarks of a rapidly spinning black hole. In this sense, the images from the Event Horizon Telescope (EHT) do confirm general relativity.



            The comparisons between general relativity's predictions and the observed images are described in detail in the fifth event horizon telescope paper (L$5$ in [$1$]), and some of them have already been reviewed on Physics SE:



            • Why isn't the circumferential light around the M87 black hole's event horizon symmetric?


            References:



            [$1$] https://iopscience.iop.org/issue/2041-8205/875/1, Table of contents of The Astrophysical Journal Letters, volume 875, number 1 (2019 April 10), with six downloadable articles (L$1$ thorugh L$6$)



            [$2$] https://www.nasa.gov/feature/goddard/2017/messier-87



            [$3$] "Black Hole-Powered Jet of Electrons and Sub-Atomic Particles Streams From Center of Galaxy M87," http://hubblesite.org/image/968/news_release/2000-20



            [$4$] "Formation of the radio jet in M87 at 100 Schwarzschild radii from the central black hole," Nature 401, 891-892 (1999), https://www.nature.com/articles/44780



            [$5$] "A precise measurement of the magnetic field in the corona of the black hole binary V404 Cygni," Science 358: 1299-1302 (2017), https://science.sciencemag.org/content/358/6368/1299



            [$6$] "Fluorescent iron lines as a probe of astrophysical black hole systems," https://arxiv.org/abs/astro-ph/0212065



            [$7$] Broderick et al, "The Event Horizon of M87," https://arxiv.org/abs/1503.03873






            share|cite|improve this answer











            $endgroup$



            Answer: The images from the Event Horizon Telescope (EHT) are consistent with what general relativity predicts. So if that's what the OP means by "confirm general relativity", then the answer is yes.



            To appreciate the significance of the EHT images, we need to remember how science works. Theories are inspired by observations, but theories are not deduced from observations; we certainly cannot deduce general relativity from a single image. It's the other way around: theories make predictions. If the predictions consistently match what we observe, then we say that the theory works. Of course, contriving a theory that makes one prediction that matches one observation is trivial. Finding a theory that makes many predictions that match many observations is more challenging. General relativity is such a theory, and its agreement with this new observation — the images from the Event Horizon Telescope — is a nice addition to the large portfolio of general relativity's confirmed predictions.



            Even though the black hole is enormous, it is so far away that the diameter of the imaged ring spans less than $20$ billionths of a degree ($<20times 10^-9$ degree) in the sky, so pristine resolution cannot be expected; the fact that they were able to resolve it at all is remarkable. Still, the image shows some general features that are consistent with what is expected from the light-bending effects associated with a rapidly spinning black hole in general relativity — not just any rapidly spinning black hole, but one whose size, mass, spin, and orientation are all consistent with other observations associated with that same black hole in the core of the galaxy M87.



            A few of these observations are reviewed below, followed by comments on how the EHT images compare to predictions from general relativity.




            Other observations: The jet



            One of the most prominent associated observations is the jet emanating from the galaxy's core, shown here in images from the Hubble Space Telescope [$2$]:



            enter image description here



            To give a feeling for the scale of this picture, this is what hubblesite.org [$3$] says about the image:




            At a distance of 50 million light-years, M87 is too distant for Hubble to discern individual stars. The dozens of star-like points swarming about M87 are, instead, themselves clusters of hundreds of thousands of stars each.




            Here's another view of the jet, with scale-bars:



            enter image description here



            This image (from figure 2 in [$4$]) was made in 1999 using VLBI observations at a wavelength of 7 millimeters. The white dot marked $6r_S$ represents a circle with a diameter of $6$ times the alleged Schwarzschild radius. The scale bar marked "$1$ kpc" represents one kiloparsec, which is roughly 3000 light-years.



            According to general relativity, a rapidly spinning black hole with an accretion disk can generate intense magnetic fields (but see [$5$]) that funnel material from the accreting plasma into a jet emanating along the black hole's axis of rotation. The fact that the observed jet is so straight over a distance of thousands of light-years implies that it must be produced by an engine that maintains a very consistent orientation for a time span of at least thousands of years, as a supermassive black hole is expected to do.




            Other observations: The accretion disk



            According to [$6$]:




            HST [Hubble Space Telescope] imaged a disk of ionized gas, with a radius of $sim$ 50 pc [50 parsecs, roughly 150 light-years] centered on the galactic core... The high resolution of HST allowed the spectrum [which is sensitive to the Doppler effect] of this ionized gas to be measured as a function of position across the gas disk, thereby allowing the kinematics of the disk to be determined... It was found that the velocity profile of the central 20 pc of the gas disk possessed a Keplerian profile (i.e., $v propto r^-1/2$) as expected if the gas was orbiting in the gravitational potential of a point-like mass... The only known and long-lived object to possess such a large mass in a small region of space, and be as under-luminous as observed, is a SMBH [Super-Massive Black Hole].




            In other words, these observations showed evidence for gas disk with the velocity profile that would be expected if it were orbiting a supermassive black hole. Note that the measured gas velocities on opposite sides of the central body differ from each other by roughly 1000 kilometers per second.




            Other observations: The absense of strong surface emission



            According to a report [$7$] published in 2015:




            Observations at millimeter wavelengths with the Event Horizon Telescope have localized the emission from the base of this jet [shown above] to angular scales comparable to the putative black hole horizon. The jet might be powered directly by an accretion disk or by electromagnetic extraction of the rotational energy of the black hole. However, even the latter mechanism requires a confining thick accretion disk to maintain the required magnetic flux near the black hole. Therefore, regardless of the jet mechanism, the observed jet power in M87 implies a certain minimum mass accretion rate. If the central compact object in M87 were not a black hole but had a surface, this accretion would result in considerable thermal near-infrared and optical emission from the surface. Current flux limits on the nucleus of M87 strongly constrain any such surface emission. This rules out the presence of a surface and thereby provides indirect evidence for an event horizon.




            Regarding why the event horizon of a black hole is expected to be so dim even though the intense fields generate a powerful jet, see these Physics SE posts:



            • How can Quasars emit anything if they're black holes?


            • How can cosmic jets exist?



            Comparing the EHT images to predictions



            Page 5 in the first event horizon telescope paper (L$1$ in [$1$]) says:




            The appearance of M87* has been modeled successfully using GRMHD [general-relativistic magnetohydrodynamics] simulations, which describe a turbulent, hot, magnetized disk orbiting a Kerr black hole. They naturally produce a powerful jet and can explain the broadband spectral energy distribution observed in LLAGNs. At a wavelength of 1.3mm, and as observed here, the simulations also predict a shadow and an asymmetric emission ring.




            Page 6 says:




            ...adopting an inclination of $17^circ$ between the approaching jet and the line of sight..., the west orientation of the jet, and a corotating disk model, matter in the bottom part of the image is moving toward the observer (clockwise rotation as seen from Earth). This is consistent with the rotation of the ionized gas on scales of 20 pc [20 parsecs, roughly 60 light-years], i.e., 7000 $r_g$ ["where $r_gequiv GM/c^2$ is the characteristic lengthscale of a black hole"]... and with the inferred sense of rotation from VLBI observations at 7 mm...




            These excerpts say that when using black-hole parameters consistent with other observations, general relativity can predict the features of the images observed by the EHT. These features, including the reduced brightness in the center and the asymmetry of the brightness of the ring, with an orientation consistent with the observed jet, are hallmarks of a rapidly spinning black hole. In this sense, the images from the Event Horizon Telescope (EHT) do confirm general relativity.



            The comparisons between general relativity's predictions and the observed images are described in detail in the fifth event horizon telescope paper (L$5$ in [$1$]), and some of them have already been reviewed on Physics SE:



            • Why isn't the circumferential light around the M87 black hole's event horizon symmetric?


            References:



            [$1$] https://iopscience.iop.org/issue/2041-8205/875/1, Table of contents of The Astrophysical Journal Letters, volume 875, number 1 (2019 April 10), with six downloadable articles (L$1$ thorugh L$6$)



            [$2$] https://www.nasa.gov/feature/goddard/2017/messier-87



            [$3$] "Black Hole-Powered Jet of Electrons and Sub-Atomic Particles Streams From Center of Galaxy M87," http://hubblesite.org/image/968/news_release/2000-20



            [$4$] "Formation of the radio jet in M87 at 100 Schwarzschild radii from the central black hole," Nature 401, 891-892 (1999), https://www.nature.com/articles/44780



            [$5$] "A precise measurement of the magnetic field in the corona of the black hole binary V404 Cygni," Science 358: 1299-1302 (2017), https://science.sciencemag.org/content/358/6368/1299



            [$6$] "Fluorescent iron lines as a probe of astrophysical black hole systems," https://arxiv.org/abs/astro-ph/0212065



            [$7$] Broderick et al, "The Event Horizon of M87," https://arxiv.org/abs/1503.03873







            share|cite|improve this answer














            share|cite|improve this answer



            share|cite|improve this answer








            edited 2 days ago

























            answered 2 days ago









            Chiral AnomalyChiral Anomaly

            13.9k21946




            13.9k21946





















                12












                $begingroup$

                If you google "m87 and general relativity" you get a list and videos on confirmation.



                This is an exaggerated response to an interesting "photograph", because it looks just like what has been calculated using the theory of general relativity for black holes.



                General relativity has been confirmed by many cosmological observations, including the calculations for the GPS signal and black holes were proposed within the framework of General relativity by Karl Schwarzschild . It is very interesting that the image developed exactly in the topology predicted by the GR equations, but the validation of GR did not really depend on this. (If a funny topology not predicted had been seen it would actually be more interesting because it would have to be modeled by something more complicated than a Kerr black hole., and maybe a modification to GR might have been proposed) .



                So the image is consistent with the expectation of a Kerr black hole, and in this sense it validates General Relativity.






                share|cite|improve this answer









                $endgroup$












                • $begingroup$
                  Anna thank you . But if say so , you mean GR is what final theory of gravity or is it simply the best of we have explaining everything that come ahead. But I have very acute problem knowing why didn't he tell us about why exactly space curves
                  $endgroup$
                  – Liquid
                  2 days ago






                • 8




                  $begingroup$
                  GR is the theory that fits our observations up to now. There are theorists trying to propose modification to GR. Well, in physics we cannot answer "Why" questions, but "how" from certain postulates using some equations we can fit observations. The "why these postulates and equations" belongs to metaphysics, not physics. Einstein was a physicist.
                  $endgroup$
                  – anna v
                  2 days ago










                • $begingroup$
                  @annav: I think the kind of "why" Liquid was talking about is less a matter of metaphysics and more in the sense of "why do these two pieces of iron seem to attract one another?"
                  $endgroup$
                  – Nicol Bolas
                  2 days ago










                • $begingroup$
                  There is no "final theory" in science. All of them are "the best we have".
                  $endgroup$
                  – nasu
                  2 days ago










                • $begingroup$
                  @NicolBolas yes . I mean it
                  $endgroup$
                  – Liquid
                  yesterday















                12












                $begingroup$

                If you google "m87 and general relativity" you get a list and videos on confirmation.



                This is an exaggerated response to an interesting "photograph", because it looks just like what has been calculated using the theory of general relativity for black holes.



                General relativity has been confirmed by many cosmological observations, including the calculations for the GPS signal and black holes were proposed within the framework of General relativity by Karl Schwarzschild . It is very interesting that the image developed exactly in the topology predicted by the GR equations, but the validation of GR did not really depend on this. (If a funny topology not predicted had been seen it would actually be more interesting because it would have to be modeled by something more complicated than a Kerr black hole., and maybe a modification to GR might have been proposed) .



                So the image is consistent with the expectation of a Kerr black hole, and in this sense it validates General Relativity.






                share|cite|improve this answer









                $endgroup$












                • $begingroup$
                  Anna thank you . But if say so , you mean GR is what final theory of gravity or is it simply the best of we have explaining everything that come ahead. But I have very acute problem knowing why didn't he tell us about why exactly space curves
                  $endgroup$
                  – Liquid
                  2 days ago






                • 8




                  $begingroup$
                  GR is the theory that fits our observations up to now. There are theorists trying to propose modification to GR. Well, in physics we cannot answer "Why" questions, but "how" from certain postulates using some equations we can fit observations. The "why these postulates and equations" belongs to metaphysics, not physics. Einstein was a physicist.
                  $endgroup$
                  – anna v
                  2 days ago










                • $begingroup$
                  @annav: I think the kind of "why" Liquid was talking about is less a matter of metaphysics and more in the sense of "why do these two pieces of iron seem to attract one another?"
                  $endgroup$
                  – Nicol Bolas
                  2 days ago










                • $begingroup$
                  There is no "final theory" in science. All of them are "the best we have".
                  $endgroup$
                  – nasu
                  2 days ago










                • $begingroup$
                  @NicolBolas yes . I mean it
                  $endgroup$
                  – Liquid
                  yesterday













                12












                12








                12





                $begingroup$

                If you google "m87 and general relativity" you get a list and videos on confirmation.



                This is an exaggerated response to an interesting "photograph", because it looks just like what has been calculated using the theory of general relativity for black holes.



                General relativity has been confirmed by many cosmological observations, including the calculations for the GPS signal and black holes were proposed within the framework of General relativity by Karl Schwarzschild . It is very interesting that the image developed exactly in the topology predicted by the GR equations, but the validation of GR did not really depend on this. (If a funny topology not predicted had been seen it would actually be more interesting because it would have to be modeled by something more complicated than a Kerr black hole., and maybe a modification to GR might have been proposed) .



                So the image is consistent with the expectation of a Kerr black hole, and in this sense it validates General Relativity.






                share|cite|improve this answer









                $endgroup$



                If you google "m87 and general relativity" you get a list and videos on confirmation.



                This is an exaggerated response to an interesting "photograph", because it looks just like what has been calculated using the theory of general relativity for black holes.



                General relativity has been confirmed by many cosmological observations, including the calculations for the GPS signal and black holes were proposed within the framework of General relativity by Karl Schwarzschild . It is very interesting that the image developed exactly in the topology predicted by the GR equations, but the validation of GR did not really depend on this. (If a funny topology not predicted had been seen it would actually be more interesting because it would have to be modeled by something more complicated than a Kerr black hole., and maybe a modification to GR might have been proposed) .



                So the image is consistent with the expectation of a Kerr black hole, and in this sense it validates General Relativity.







                share|cite|improve this answer












                share|cite|improve this answer



                share|cite|improve this answer










                answered 2 days ago









                anna vanna v

                162k8153456




                162k8153456











                • $begingroup$
                  Anna thank you . But if say so , you mean GR is what final theory of gravity or is it simply the best of we have explaining everything that come ahead. But I have very acute problem knowing why didn't he tell us about why exactly space curves
                  $endgroup$
                  – Liquid
                  2 days ago






                • 8




                  $begingroup$
                  GR is the theory that fits our observations up to now. There are theorists trying to propose modification to GR. Well, in physics we cannot answer "Why" questions, but "how" from certain postulates using some equations we can fit observations. The "why these postulates and equations" belongs to metaphysics, not physics. Einstein was a physicist.
                  $endgroup$
                  – anna v
                  2 days ago










                • $begingroup$
                  @annav: I think the kind of "why" Liquid was talking about is less a matter of metaphysics and more in the sense of "why do these two pieces of iron seem to attract one another?"
                  $endgroup$
                  – Nicol Bolas
                  2 days ago










                • $begingroup$
                  There is no "final theory" in science. All of them are "the best we have".
                  $endgroup$
                  – nasu
                  2 days ago










                • $begingroup$
                  @NicolBolas yes . I mean it
                  $endgroup$
                  – Liquid
                  yesterday
















                • $begingroup$
                  Anna thank you . But if say so , you mean GR is what final theory of gravity or is it simply the best of we have explaining everything that come ahead. But I have very acute problem knowing why didn't he tell us about why exactly space curves
                  $endgroup$
                  – Liquid
                  2 days ago






                • 8




                  $begingroup$
                  GR is the theory that fits our observations up to now. There are theorists trying to propose modification to GR. Well, in physics we cannot answer "Why" questions, but "how" from certain postulates using some equations we can fit observations. The "why these postulates and equations" belongs to metaphysics, not physics. Einstein was a physicist.
                  $endgroup$
                  – anna v
                  2 days ago










                • $begingroup$
                  @annav: I think the kind of "why" Liquid was talking about is less a matter of metaphysics and more in the sense of "why do these two pieces of iron seem to attract one another?"
                  $endgroup$
                  – Nicol Bolas
                  2 days ago










                • $begingroup$
                  There is no "final theory" in science. All of them are "the best we have".
                  $endgroup$
                  – nasu
                  2 days ago










                • $begingroup$
                  @NicolBolas yes . I mean it
                  $endgroup$
                  – Liquid
                  yesterday















                $begingroup$
                Anna thank you . But if say so , you mean GR is what final theory of gravity or is it simply the best of we have explaining everything that come ahead. But I have very acute problem knowing why didn't he tell us about why exactly space curves
                $endgroup$
                – Liquid
                2 days ago




                $begingroup$
                Anna thank you . But if say so , you mean GR is what final theory of gravity or is it simply the best of we have explaining everything that come ahead. But I have very acute problem knowing why didn't he tell us about why exactly space curves
                $endgroup$
                – Liquid
                2 days ago




                8




                8




                $begingroup$
                GR is the theory that fits our observations up to now. There are theorists trying to propose modification to GR. Well, in physics we cannot answer "Why" questions, but "how" from certain postulates using some equations we can fit observations. The "why these postulates and equations" belongs to metaphysics, not physics. Einstein was a physicist.
                $endgroup$
                – anna v
                2 days ago




                $begingroup$
                GR is the theory that fits our observations up to now. There are theorists trying to propose modification to GR. Well, in physics we cannot answer "Why" questions, but "how" from certain postulates using some equations we can fit observations. The "why these postulates and equations" belongs to metaphysics, not physics. Einstein was a physicist.
                $endgroup$
                – anna v
                2 days ago












                $begingroup$
                @annav: I think the kind of "why" Liquid was talking about is less a matter of metaphysics and more in the sense of "why do these two pieces of iron seem to attract one another?"
                $endgroup$
                – Nicol Bolas
                2 days ago




                $begingroup$
                @annav: I think the kind of "why" Liquid was talking about is less a matter of metaphysics and more in the sense of "why do these two pieces of iron seem to attract one another?"
                $endgroup$
                – Nicol Bolas
                2 days ago












                $begingroup$
                There is no "final theory" in science. All of them are "the best we have".
                $endgroup$
                – nasu
                2 days ago




                $begingroup$
                There is no "final theory" in science. All of them are "the best we have".
                $endgroup$
                – nasu
                2 days ago












                $begingroup$
                @NicolBolas yes . I mean it
                $endgroup$
                – Liquid
                yesterday




                $begingroup$
                @NicolBolas yes . I mean it
                $endgroup$
                – Liquid
                yesterday











                9












                $begingroup$

                There are two main aspects to the observations that are consistent with the predictions of GR and where the measurements stood a realistic chance of falsifying GR (and that is all you can do - design experiments capable of falsifying theories).



                The first is that the bright photon ring radius is to within about 10% of the prediction of GR for a black hole with a mass independently determined from the motions of stars in the central regions of M87.



                The second is that the black hole "shadow" is almost circular, to better than 10%. Again, this is predicted by GR in all but some extreme combinations of spin and spin-axis orientation. The maximum non circularity of the shadow caused by a Kerr black hole is 10% (see section 9.5 of the fifth Event Horizon Telescope M87 papers). Other explanations for the space-time distortion could have resulted in more oblate/prolate shadows.






                share|cite|improve this answer











                $endgroup$

















                  9












                  $begingroup$

                  There are two main aspects to the observations that are consistent with the predictions of GR and where the measurements stood a realistic chance of falsifying GR (and that is all you can do - design experiments capable of falsifying theories).



                  The first is that the bright photon ring radius is to within about 10% of the prediction of GR for a black hole with a mass independently determined from the motions of stars in the central regions of M87.



                  The second is that the black hole "shadow" is almost circular, to better than 10%. Again, this is predicted by GR in all but some extreme combinations of spin and spin-axis orientation. The maximum non circularity of the shadow caused by a Kerr black hole is 10% (see section 9.5 of the fifth Event Horizon Telescope M87 papers). Other explanations for the space-time distortion could have resulted in more oblate/prolate shadows.






                  share|cite|improve this answer











                  $endgroup$















                    9












                    9








                    9





                    $begingroup$

                    There are two main aspects to the observations that are consistent with the predictions of GR and where the measurements stood a realistic chance of falsifying GR (and that is all you can do - design experiments capable of falsifying theories).



                    The first is that the bright photon ring radius is to within about 10% of the prediction of GR for a black hole with a mass independently determined from the motions of stars in the central regions of M87.



                    The second is that the black hole "shadow" is almost circular, to better than 10%. Again, this is predicted by GR in all but some extreme combinations of spin and spin-axis orientation. The maximum non circularity of the shadow caused by a Kerr black hole is 10% (see section 9.5 of the fifth Event Horizon Telescope M87 papers). Other explanations for the space-time distortion could have resulted in more oblate/prolate shadows.






                    share|cite|improve this answer











                    $endgroup$



                    There are two main aspects to the observations that are consistent with the predictions of GR and where the measurements stood a realistic chance of falsifying GR (and that is all you can do - design experiments capable of falsifying theories).



                    The first is that the bright photon ring radius is to within about 10% of the prediction of GR for a black hole with a mass independently determined from the motions of stars in the central regions of M87.



                    The second is that the black hole "shadow" is almost circular, to better than 10%. Again, this is predicted by GR in all but some extreme combinations of spin and spin-axis orientation. The maximum non circularity of the shadow caused by a Kerr black hole is 10% (see section 9.5 of the fifth Event Horizon Telescope M87 papers). Other explanations for the space-time distortion could have resulted in more oblate/prolate shadows.







                    share|cite|improve this answer














                    share|cite|improve this answer



                    share|cite|improve this answer








                    edited yesterday

























                    answered yesterday









                    Rob JeffriesRob Jeffries

                    71.4k7152248




                    71.4k7152248





















                        6












                        $begingroup$

                        As some other answers say, the question "does this confirm General Relativity" gets it the wrong way round.



                        GR is our current best understanding of how mass distorts spacetime, giving rise to many phenomenae we see around us. GR explains these as a result of distorted spacetime. It's incredibly accurate and has been tested in many, many ways.



                        Without it, everyday things like satnav/GPS couldn't work, so it's also directly relevant to daily life. (GPS is so sensitive to timing that its calculations have to allow for the effects of General and Special Relativity)



                        But because it's only our current best understanding, it's also likely and expected that GR will be eventually found to be "wrong enough on some things" that it will be superseded by some better theory in future. This happened to Newton's Laws of Motion - they are good enough for many parts of daily life, but we now know they aren't accurate in a lot of situations that exist, in which Special and General Relativity are more accurate.



                        Why tests are important



                        Because GR is our current "best theory", and relied on a lot, it is useful to know how accurately it matches experimental results, for at least 6 reasons:



                        • to find (or double check) where we can rely on it, and if there are any places we can't rely on it. (This is especially valuable if the test was of of a kind that hasn't been done before, or if the test acts as a double-check on some existing test that hasn't been reproduced before).

                        • to find where it seems to predict something which doesn't match what we see - this might suggest areas where we can look for "new physics" or unknown knowledge about the universe we are in, or alternatively, it can suggest how to improve in the care needed by experimenters and theorists, to avoid misinterpretations

                        • to tell us if our experimental technique is sound, and whether a particular technique is capable of being used, or if it's working properly

                        • For general education about science (in some high profile cases)

                        • to explain to governments that their funding is useful and produces valuable knowledge. Funding is needed but must compete with other national needs, so showing results is useful.

                        • Some tests require developing whole new methods, algorithms, computer technology, and analytic techniques, which then have wider benefits. For example, the Large Hadron Collider required development of entire data storage and distributed networking methods which have been used in cloud computing and the wider computing world.

                        The Black Hole photograph's importance



                        In that sense, the imaging of a black hole is important. That isn't because it 'proves" GR is correct (it can't), but because of these same 6 reasons:




                        • it confirms that GR is reliable enough that what it predicts a Black Hole should look like, if we could ever photograph one, really does seem to match what we see when we do actually photograph one from billions of light years away. Even with the naked eye, we can see the accretion disk, the shadow, the black centre, and the effects of rotation and temperature on the bright areas. Probably when the image is analysed very closely, it might also confirm that we see many other things that match what GR said we should expect, which aren't easy to see with the naked eye. The photograph could have looked very different from the prediction, but it didn't.
                          It is also the first test of its kind.


                        • it may contain extra data that when studied, lets us improve current theories about the universe (and the lives of objects within it), or helps us to narrow down what theories and parameters can be "correct"


                        • it tells us that our technique of ultra long range photography using this new technique is sound, so we can move forward and improve it, or use it to view other objects directly.

                        • it is dramatic, so many people will read it and learn more about the universe.
                          So it can be educational


                        • it shows that the funding aim was successfully achieved when the project was funded.


                        • it required whole new algorithms and techniques to be developed, which may be beneficial to society or in other future projects, and which can themselves be built upon and expanded.
                          Nothing like it, on this scale, had been tried before.

                        Comment



                        It's only in that sense that this experiment and the Black Hole photo "confirms" anything. But it can still only be one more piece of evidence that so far, GR isn't letting us down.



                        It doesn't change the fact that ultimately we do expect that some limit to GR's accuracy will probably be found, or we will encounter a new situation where GR doesn't apply very well. When that happens, we hope that a better theory will arise that matches GR in the areas GR works, and also matches those new findings in the areas GR doesn't work. So far, this isn't a test that shows a limit has been found, though.



                        So it's win-win really, either we prove our existing theories work in an even more extreme or novel case, or we get a hint of an unexpected difference which will help us to understand things better (either improving our tests to remove a mistake, or improving our theory if the test was correct).






                        share|cite|improve this answer











                        $endgroup$

















                          6












                          $begingroup$

                          As some other answers say, the question "does this confirm General Relativity" gets it the wrong way round.



                          GR is our current best understanding of how mass distorts spacetime, giving rise to many phenomenae we see around us. GR explains these as a result of distorted spacetime. It's incredibly accurate and has been tested in many, many ways.



                          Without it, everyday things like satnav/GPS couldn't work, so it's also directly relevant to daily life. (GPS is so sensitive to timing that its calculations have to allow for the effects of General and Special Relativity)



                          But because it's only our current best understanding, it's also likely and expected that GR will be eventually found to be "wrong enough on some things" that it will be superseded by some better theory in future. This happened to Newton's Laws of Motion - they are good enough for many parts of daily life, but we now know they aren't accurate in a lot of situations that exist, in which Special and General Relativity are more accurate.



                          Why tests are important



                          Because GR is our current "best theory", and relied on a lot, it is useful to know how accurately it matches experimental results, for at least 6 reasons:



                          • to find (or double check) where we can rely on it, and if there are any places we can't rely on it. (This is especially valuable if the test was of of a kind that hasn't been done before, or if the test acts as a double-check on some existing test that hasn't been reproduced before).

                          • to find where it seems to predict something which doesn't match what we see - this might suggest areas where we can look for "new physics" or unknown knowledge about the universe we are in, or alternatively, it can suggest how to improve in the care needed by experimenters and theorists, to avoid misinterpretations

                          • to tell us if our experimental technique is sound, and whether a particular technique is capable of being used, or if it's working properly

                          • For general education about science (in some high profile cases)

                          • to explain to governments that their funding is useful and produces valuable knowledge. Funding is needed but must compete with other national needs, so showing results is useful.

                          • Some tests require developing whole new methods, algorithms, computer technology, and analytic techniques, which then have wider benefits. For example, the Large Hadron Collider required development of entire data storage and distributed networking methods which have been used in cloud computing and the wider computing world.

                          The Black Hole photograph's importance



                          In that sense, the imaging of a black hole is important. That isn't because it 'proves" GR is correct (it can't), but because of these same 6 reasons:




                          • it confirms that GR is reliable enough that what it predicts a Black Hole should look like, if we could ever photograph one, really does seem to match what we see when we do actually photograph one from billions of light years away. Even with the naked eye, we can see the accretion disk, the shadow, the black centre, and the effects of rotation and temperature on the bright areas. Probably when the image is analysed very closely, it might also confirm that we see many other things that match what GR said we should expect, which aren't easy to see with the naked eye. The photograph could have looked very different from the prediction, but it didn't.
                            It is also the first test of its kind.


                          • it may contain extra data that when studied, lets us improve current theories about the universe (and the lives of objects within it), or helps us to narrow down what theories and parameters can be "correct"


                          • it tells us that our technique of ultra long range photography using this new technique is sound, so we can move forward and improve it, or use it to view other objects directly.

                          • it is dramatic, so many people will read it and learn more about the universe.
                            So it can be educational


                          • it shows that the funding aim was successfully achieved when the project was funded.


                          • it required whole new algorithms and techniques to be developed, which may be beneficial to society or in other future projects, and which can themselves be built upon and expanded.
                            Nothing like it, on this scale, had been tried before.

                          Comment



                          It's only in that sense that this experiment and the Black Hole photo "confirms" anything. But it can still only be one more piece of evidence that so far, GR isn't letting us down.



                          It doesn't change the fact that ultimately we do expect that some limit to GR's accuracy will probably be found, or we will encounter a new situation where GR doesn't apply very well. When that happens, we hope that a better theory will arise that matches GR in the areas GR works, and also matches those new findings in the areas GR doesn't work. So far, this isn't a test that shows a limit has been found, though.



                          So it's win-win really, either we prove our existing theories work in an even more extreme or novel case, or we get a hint of an unexpected difference which will help us to understand things better (either improving our tests to remove a mistake, or improving our theory if the test was correct).






                          share|cite|improve this answer











                          $endgroup$















                            6












                            6








                            6





                            $begingroup$

                            As some other answers say, the question "does this confirm General Relativity" gets it the wrong way round.



                            GR is our current best understanding of how mass distorts spacetime, giving rise to many phenomenae we see around us. GR explains these as a result of distorted spacetime. It's incredibly accurate and has been tested in many, many ways.



                            Without it, everyday things like satnav/GPS couldn't work, so it's also directly relevant to daily life. (GPS is so sensitive to timing that its calculations have to allow for the effects of General and Special Relativity)



                            But because it's only our current best understanding, it's also likely and expected that GR will be eventually found to be "wrong enough on some things" that it will be superseded by some better theory in future. This happened to Newton's Laws of Motion - they are good enough for many parts of daily life, but we now know they aren't accurate in a lot of situations that exist, in which Special and General Relativity are more accurate.



                            Why tests are important



                            Because GR is our current "best theory", and relied on a lot, it is useful to know how accurately it matches experimental results, for at least 6 reasons:



                            • to find (or double check) where we can rely on it, and if there are any places we can't rely on it. (This is especially valuable if the test was of of a kind that hasn't been done before, or if the test acts as a double-check on some existing test that hasn't been reproduced before).

                            • to find where it seems to predict something which doesn't match what we see - this might suggest areas where we can look for "new physics" or unknown knowledge about the universe we are in, or alternatively, it can suggest how to improve in the care needed by experimenters and theorists, to avoid misinterpretations

                            • to tell us if our experimental technique is sound, and whether a particular technique is capable of being used, or if it's working properly

                            • For general education about science (in some high profile cases)

                            • to explain to governments that their funding is useful and produces valuable knowledge. Funding is needed but must compete with other national needs, so showing results is useful.

                            • Some tests require developing whole new methods, algorithms, computer technology, and analytic techniques, which then have wider benefits. For example, the Large Hadron Collider required development of entire data storage and distributed networking methods which have been used in cloud computing and the wider computing world.

                            The Black Hole photograph's importance



                            In that sense, the imaging of a black hole is important. That isn't because it 'proves" GR is correct (it can't), but because of these same 6 reasons:




                            • it confirms that GR is reliable enough that what it predicts a Black Hole should look like, if we could ever photograph one, really does seem to match what we see when we do actually photograph one from billions of light years away. Even with the naked eye, we can see the accretion disk, the shadow, the black centre, and the effects of rotation and temperature on the bright areas. Probably when the image is analysed very closely, it might also confirm that we see many other things that match what GR said we should expect, which aren't easy to see with the naked eye. The photograph could have looked very different from the prediction, but it didn't.
                              It is also the first test of its kind.


                            • it may contain extra data that when studied, lets us improve current theories about the universe (and the lives of objects within it), or helps us to narrow down what theories and parameters can be "correct"


                            • it tells us that our technique of ultra long range photography using this new technique is sound, so we can move forward and improve it, or use it to view other objects directly.

                            • it is dramatic, so many people will read it and learn more about the universe.
                              So it can be educational


                            • it shows that the funding aim was successfully achieved when the project was funded.


                            • it required whole new algorithms and techniques to be developed, which may be beneficial to society or in other future projects, and which can themselves be built upon and expanded.
                              Nothing like it, on this scale, had been tried before.

                            Comment



                            It's only in that sense that this experiment and the Black Hole photo "confirms" anything. But it can still only be one more piece of evidence that so far, GR isn't letting us down.



                            It doesn't change the fact that ultimately we do expect that some limit to GR's accuracy will probably be found, or we will encounter a new situation where GR doesn't apply very well. When that happens, we hope that a better theory will arise that matches GR in the areas GR works, and also matches those new findings in the areas GR doesn't work. So far, this isn't a test that shows a limit has been found, though.



                            So it's win-win really, either we prove our existing theories work in an even more extreme or novel case, or we get a hint of an unexpected difference which will help us to understand things better (either improving our tests to remove a mistake, or improving our theory if the test was correct).






                            share|cite|improve this answer











                            $endgroup$



                            As some other answers say, the question "does this confirm General Relativity" gets it the wrong way round.



                            GR is our current best understanding of how mass distorts spacetime, giving rise to many phenomenae we see around us. GR explains these as a result of distorted spacetime. It's incredibly accurate and has been tested in many, many ways.



                            Without it, everyday things like satnav/GPS couldn't work, so it's also directly relevant to daily life. (GPS is so sensitive to timing that its calculations have to allow for the effects of General and Special Relativity)



                            But because it's only our current best understanding, it's also likely and expected that GR will be eventually found to be "wrong enough on some things" that it will be superseded by some better theory in future. This happened to Newton's Laws of Motion - they are good enough for many parts of daily life, but we now know they aren't accurate in a lot of situations that exist, in which Special and General Relativity are more accurate.



                            Why tests are important



                            Because GR is our current "best theory", and relied on a lot, it is useful to know how accurately it matches experimental results, for at least 6 reasons:



                            • to find (or double check) where we can rely on it, and if there are any places we can't rely on it. (This is especially valuable if the test was of of a kind that hasn't been done before, or if the test acts as a double-check on some existing test that hasn't been reproduced before).

                            • to find where it seems to predict something which doesn't match what we see - this might suggest areas where we can look for "new physics" or unknown knowledge about the universe we are in, or alternatively, it can suggest how to improve in the care needed by experimenters and theorists, to avoid misinterpretations

                            • to tell us if our experimental technique is sound, and whether a particular technique is capable of being used, or if it's working properly

                            • For general education about science (in some high profile cases)

                            • to explain to governments that their funding is useful and produces valuable knowledge. Funding is needed but must compete with other national needs, so showing results is useful.

                            • Some tests require developing whole new methods, algorithms, computer technology, and analytic techniques, which then have wider benefits. For example, the Large Hadron Collider required development of entire data storage and distributed networking methods which have been used in cloud computing and the wider computing world.

                            The Black Hole photograph's importance



                            In that sense, the imaging of a black hole is important. That isn't because it 'proves" GR is correct (it can't), but because of these same 6 reasons:




                            • it confirms that GR is reliable enough that what it predicts a Black Hole should look like, if we could ever photograph one, really does seem to match what we see when we do actually photograph one from billions of light years away. Even with the naked eye, we can see the accretion disk, the shadow, the black centre, and the effects of rotation and temperature on the bright areas. Probably when the image is analysed very closely, it might also confirm that we see many other things that match what GR said we should expect, which aren't easy to see with the naked eye. The photograph could have looked very different from the prediction, but it didn't.
                              It is also the first test of its kind.


                            • it may contain extra data that when studied, lets us improve current theories about the universe (and the lives of objects within it), or helps us to narrow down what theories and parameters can be "correct"


                            • it tells us that our technique of ultra long range photography using this new technique is sound, so we can move forward and improve it, or use it to view other objects directly.

                            • it is dramatic, so many people will read it and learn more about the universe.
                              So it can be educational


                            • it shows that the funding aim was successfully achieved when the project was funded.


                            • it required whole new algorithms and techniques to be developed, which may be beneficial to society or in other future projects, and which can themselves be built upon and expanded.
                              Nothing like it, on this scale, had been tried before.

                            Comment



                            It's only in that sense that this experiment and the Black Hole photo "confirms" anything. But it can still only be one more piece of evidence that so far, GR isn't letting us down.



                            It doesn't change the fact that ultimately we do expect that some limit to GR's accuracy will probably be found, or we will encounter a new situation where GR doesn't apply very well. When that happens, we hope that a better theory will arise that matches GR in the areas GR works, and also matches those new findings in the areas GR doesn't work. So far, this isn't a test that shows a limit has been found, though.



                            So it's win-win really, either we prove our existing theories work in an even more extreme or novel case, or we get a hint of an unexpected difference which will help us to understand things better (either improving our tests to remove a mistake, or improving our theory if the test was correct).







                            share|cite|improve this answer














                            share|cite|improve this answer



                            share|cite|improve this answer








                            edited yesterday

























                            answered yesterday









                            StilezStilez

                            1,528413




                            1,528413



























                                draft saved

                                draft discarded
















































                                Thanks for contributing an answer to Physics Stack Exchange!


                                • Please be sure to answer the question. Provide details and share your research!

                                But avoid


                                • Asking for help, clarification, or responding to other answers.

                                • Making statements based on opinion; back them up with references or personal experience.

                                Use MathJax to format equations. MathJax reference.


                                To learn more, see our tips on writing great answers.




                                draft saved


                                draft discarded














                                StackExchange.ready(
                                function ()
                                StackExchange.openid.initPostLogin('.new-post-login', 'https%3a%2f%2fphysics.stackexchange.com%2fquestions%2f472323%2fdid-the-new-image-of-black-hole-confirm-the-general-theory-of-relativity%23new-answer', 'question_page');

                                );

                                Post as a guest















                                Required, but never shown





















































                                Required, but never shown














                                Required, but never shown












                                Required, but never shown







                                Required, but never shown

































                                Required, but never shown














                                Required, but never shown












                                Required, but never shown







                                Required, but never shown







                                Popular posts from this blog

                                Get product attribute by attribute group code in magento 2get product attribute by product attribute group in magento 2Magento 2 Log Bundle Product Data in List Page?How to get all product attribute of a attribute group of Default attribute set?Magento 2.1 Create a filter in the product grid by new attributeMagento 2 : Get Product Attribute values By GroupMagento 2 How to get all existing values for one attributeMagento 2 get custom attribute of a single product inside a pluginMagento 2.3 How to get all the Multi Source Inventory (MSI) locations collection in custom module?Magento2: how to develop rest API to get new productsGet product attribute by attribute group code ( [attribute_group_code] ) in magento 2

                                Category:9 (number) SubcategoriesMedia in category "9 (number)"Navigation menuUpload mediaGND ID: 4485639-8Library of Congress authority ID: sh85091979ReasonatorScholiaStatistics

                                Magento 2.3: How do i solve this, Not registered handle, on custom form?How can i rewrite TierPrice Block in Magento2magento 2 captcha not rendering if I override layout xmlmain.CRITICAL: Plugin class doesn't existMagento 2 : Problem while adding custom button order view page?Magento 2.2.5: Overriding Admin Controller sales/orderMagento 2.2.5: Add, Update and Delete existing products Custom OptionsMagento 2.3 : File Upload issue in UI Component FormMagento2 Not registered handleHow to configured Form Builder Js in my custom magento 2.3.0 module?Magento 2.3. How to create image upload field in an admin form